You are on page 1of 119

KVS RO EKM/CLASS XII MATHS/Term-2

STUDY MATERIAL
MATHEMATICS – [041]
Based on Latest CBSE Exam Pattern for
Session 2022-23

Page 1 of 119
KVSRO/EKM/XI/MATHS

KVS RO EKM/CLASS XII MATHS/Term-1

OUR PATRON
HONOURABLE DEPUTY COMMISSIONER

KVS RO ERNAKULAM REGION

SHRI R. SENTHIL KUMAR

Page 2 of 119
KVSRO/EKM/XI/MATHS

Smt.Deepti Nair
Assistant Commissioner
KVSRO, Ernakulam

Sh. Santhosh Kumar N Sh. Ajayakumar


Assistant Commissioner Assistant Commissioner
KVSRO, Ernakulam KVSRO,Ernakulam

Co-ordinated by

Sh. K P Sudhakaran
Principal
KV CRPF Peringome

Page 3 of 119
KVSRO/EKM/XI/MATHS

Page 4 of 119
KVSRO/EKM/XI/MATHS

TEAM OF TEACHERS WHO


CONTRIBUTED……..

SL.NO. NAME OF THE TEACHER NAME OF THE KV


1 Mr Sreekumar K Adoor Shift 2
2 Mr. Santhosh B Rubber board Kottayam
3 Mrs Sreedevi P G NTPC , Kayamkulam
4 Mrs Maya R , SAP , Peroorkada
5 Mrs. Letha K Nair AFS Akkulam.
6 Mrs Geetha Kumari R Kollam
7 Mrs Beena Joseph CRPF, Pallippuram
8 Mr Hari Kumar P S Pattom Shift 1
9 Mr Narasimh Prabhu NO.1 CPCRI, Kasaragod
10 Mrs. Sreeletha Vinuraj Pangode
11 Mrs. Sunitha Sanalkumar Pangode
12 Mr U Sreekumar Chenneerkara
13 Mrs. Laly Paulose Koikkara Kanjikode
14 Mrs. Ramadevi M KEPA Ramavarmapuram
15 Ms. Sathi M N INS Dronacharya

16 Mrs. Jayasree Karthikeyan INS Dronacharya


17 Mrs. Gracy Franklin No. 1 Naval Base, Kochi
18 Mrs. Mini Antony No. 1 Naval Base, Kochi

REVIEW, EDITING & COMPILATION.

SL.NO. NAME OF THE TEACHER NAME OF THE KV


1. Mr. Shanukumar R Thrissur
2. Mr. Mohankumar P No.2 NAVAL BASE, Kochi
3. Mr Harikrishnan CRPF, Peringome
4. Mrs. Bindhu Adoor Shift 1
5. Mrs Sreelekha S Kumar Ernakulam
6. Mrs. Darly John Adoor Shift 1.

Page 5 of 119
KVSRO/EKM/XI/MATHS

Content

 Syllabus for Examination 2022-23


• COURSE STRUCTURE

• COURSE CONTENT

 Chapters
• SETS

• RELATIONS & FUNCTIONS

• TRIGONOMETRIC FUNCTIONS

• COMPLEX NUMBERS & QUADRATIC EQUATIONS

• LINEAR INEQUALITIES

• PERMUTATIONS & COMBINATIONS

• BINOMIAL THEOREM

• SEQUENCE & SERIES

• STRAIGHT LINES

• CONIC SECTIONS

• INTRODUCTION TO THREE DIMENSIONAL GEOMETRY

• LIMITS & DERIVATIVES

• STATISTICS

• PROBABILITY

 Sample Question Papers & Marking Scheme

Page 6 of 119
KVSRO/EKM/XI/MATHS

MATHEMATICS

The Syllabus in the subject of Mathematics has undergone changes from time to time in accordance with growth of
the subject and emerging needs of the society. Senior Secondary stageis a launching stage from where the students
go either for higher academic education in Mathematics or for professional courses like Engineering, Physical and
Biological science, Commerce or Computer Applications. The present revised syllabus has been designed in
accordance with National Curriculum Framework 2005 and as per guidelines given in Focus Group on Teaching of
Mathematics 2005 which is to meet the emerging needs of all categories of students. Motivating the topics from
real life situations and other subject areas, greater emphasis has been laid on application of various concepts.

Objectives

The broad objectives of teaching Mathematics at senior school stage intend to help the students:

 to acquire knowledge and critical understanding, particularly by way of motivation and


visualization, of basic concepts, terms, principles, symbols and mastery of underlying
processes and skills.
 to feel the flow of reasons while proving a result or solving a problem.
 to apply the knowledge and skills acquired to solve problems and wherever possible, bymore
than one method.
 to develop positive attitude to think, analyze and articulate logically.
 to develop interest in the subject by participating in related competitions.
 to acquaint students with different aspects of Mathematics used in daily life.
 to develop an interest in students to study Mathematics as a discipline.
 to develop awareness of the need for national integration, protection of environment,
observance of small family norms, removal of social barriers, elimination of gender biases.
 to develop reverence and respect towards great Mathematicians for their contributions to
the field of Mathematics.

Page 7 of 119
KVSRO/EKM/XI/MATHS

COURSE STRUCTURE
CLASS XI (2022-23)
One Paper Total Period–240 [35 Minutes each]

Three Hours Max Marks: 80


No. Units No. of Periods Marks
I. Sets and Functions 60 23
II. Algebra 50 25
III. Coordinate Geometry 50 12
IV. Calculus 40 08
V. Statistics and Probability 40 12
Total 240 80
Internal Assessment 20

*No chapter/unit-wise weightage. Care to be taken to cover all the chapters.


Unit-I: Sets and Functions
1. Sets (20) Periods
Sets and their representations, Empty set, Finite and Infinite sets, Equal sets, Subsets, Subsets of a set of real
numbers especially intervals (with notations). Universal set. Venn diagrams. Union and Intersection of sets.
Difference of sets. Complement of a set. Properties of Complement.

2. Relations & Functions (20) Periods


Ordered pairs. Cartesian product of sets. Number of elements in the Cartesian product of two finitesets. Cartesian
product of the set of reals with itself (upto R x R x R).Definition of relation, pictorial diagrams, domain, co-domain
and range of a relation. Function as a special type of relation. Pictorial representation of a function, domain, co-
domain and range of a function. Real valued functions, domain and range of these functions, constant, identity,
polynomial, rational, modulus, signum, exponential, logarithmic and greatest integer functions, with their graphs.
Sum, difference,product and quotients of functions.

3. Trigonometric Functions (20) Periods


Positive and negative angles. Measuring angles in radians and in degrees and conversion from one measure to
another. Definition of trigonometric functions with the help of unit circle. Truth of

the identity sin2x + cos2x = 1, for all x. Signs of trigonometric functions. Domain and range of trigonometric
functions and their graphs. Expressing sin (x±y) and cos (x±y) in terms of sinx, siny,cosx & cosy and their simple

Page 8 of 119
KVSRO/EKM/XI/MATHS

applications. Deducing identities like the following


𝑥+𝑦 𝑥−𝑦
 sin 𝑥 + sin 𝑦 = 2 sin sin
2 2
𝑥+𝑦 𝑥−𝑦
 sin 𝑥 − sin 𝑦 = 2 cos sin
2 2
𝑥+𝑦 𝑥−𝑦
 cos 𝑥 + cos 𝑦 = 2 cos cos
2 2
𝑥+𝑦 𝑥−𝑦
 cos 𝑥 − cos 𝑦 = −2 sin sin
2 2

Identities related to sin2x, cos2x, tan2 x, sin3x, cos3x and tan3x.

Unit-II: Algebra
1. Complex Numbers and Quadratic Equations (10) Periods

Need for complex numbers, especially√−1, to be motivated by inability to solve some of thequadratic
equations. Algebraic properties of complex numbers. Argand plane

2. Linear Inequalities (10) Periods


Linear inequalities. Algebraic solutions of linear inequalities in one variable and their representationon the number
line.

3. Permutations and Combinations (10) Periods

Fundamental principle of counting. Factorial n. (n!) Permutations and combinations, derivation ofFormulae for nPr
and nCr and their connections, simple applications.

4. Binomial Theorem (10) Periods

Historical perspective, statement and proof of the binomial theorem for positive integral indices.Pascal’s triangle,
simple applications.

5. Sequence and Series (10) Periods

Sequence and Series. Arithmetic Mean (A.M.) Geometric Progression (G.P.), general term of aG.P., sum of n terms
of a G.P., infinite G.P. and its sum, geometric mean (G.M.), relation between
A.M. and G.M.

Unit-III: Coordinate Geometry


1. Straight Lines (15) Periods

Brief recall of two dimensional geometry from earlier classes. Slope of a line and angle between two lines. Various
forms of equations of a line: parallel to axis, point -slope form, slope-intercept form, two-point form, intercept
form, Distance of a point from a line.

Page 9 of 119
KVSRO/EKM/XI/MATHS

2. Conic Sections (25) Periods

Sections of a cone: circles, ellipse, parabola, hyperbola, a point, a straight line and a pair of intersecting lines as a
degenerated case of a conic section. Standard equations and simple properties of parabola, ellipse and hyperbola.
Standard equation of a circle.
3. Introduction to Three-dimensional Geometry (10) Periods
Coordinate axes and coordinate planes in three dimensions. Coordinates of a point. Distance between two points.

Unit-IV: Calculus

1. Limits and Derivatives (40) Periods

Derivative introduced as rate of change both as that of distance function and geometrically. Intuitive idea of limit.
Limits of polynomials and rational functions trigonometric, exponential and logarithmic functions. Definition of
derivative relate it to scope of tangent of the curve, derivative of sum, difference, product and quotient of
functions. Derivatives of polynomial and trigonometric functions.

Unit-V Statistics and Probability

1. Statistics (20) Periods

Measures of Dispersion: Range, Mean deviation, variance and standard deviation of ungrouped/grouped data.

2. Probability (20) Periods

Events; occurrence of events, ‘not’, ‘and’ and ‘or’ events, exhaustive events, mutually exclusive events, Axiomatic
(set theoretic) probability, connections with other theories of earlier classes. Probability of an event, probability of
‘not’, ‘and’ and ‘or’ events.

INTERNAL ASSESSMENT 20 MARKS


Periodic Tests ( Best 2 out of 3 tests conducted) 10 Marks
Mathematics Activities 10 Marks

Page 10 of 119
KVSRO/EKM/XI/MATHS

Chapter 1
SETS
MCQ(1 MARKS)
1. The number of subsets of a set containing n elements is

a)n b ) 2 n– 1 c) n 2 d) 2n

2. If n(A) = n (B) = 70, and n(AUB) =89,then n (A∩B) is

a) 49 b) 62 c) 51 d) 53

3. The interval form of { x: -1<x≤ 3, x € R }is

a) (-- 1 , 3 ) b) ( =1 , 3] c) [=1 , 3 ) d) [ =1 , 3]

4. Which among the following is not true

a) A 𝖴 A1 = X b) A ∩ A1 = ∅ c) A ∩ ∅=A d)A 𝖴 ∅ = A

5.Which is not a subset of {1,{2}}

a) {1} b) {2} c) {{2}} d) ∅

Short answer type questions (2 Marks)

6. IF U = {1,2,3,4,…….,10} A = {1,2,3,5} B = {2,4,6,7} , find (A – B )1.

7. If A and B are two sets and U is the universal set such that

n (U) = 700, n(A) =200, n(B) =300 and n(A∩ B )=100, find (A1 ∩ B1) .
1 23 4 5
8. Write the set A ={ , , , , } in set builder form
4 56 7 8

9. Prove that , for any two sets , (A ∩ B) 𝖴 (A – B ) = A .

10. Write the following intervals in set builder form 1) (-3 ,0 ) 2) [-23,5)

11.Find the smallest set X such that X U {1,2} = { 1,2,3,5,9}

12.Let A = { x : x € N, 0< x< 5} B = {x : x € N, 4 <x <6 } Write 1) A – B 2) B – A

13. If X = {1,2,3,4,5,6 } , A = {3,4,6} ,B = {4,5} Verify that A – B = A ∩ B1

14. Let A and B are two finite sets such that n(A ∩ B) =12, n (A-B) = 24 , n (B – A ) = 45,

find n (A ) and n(B).

15. Two finite sets have m and n elements . The number of subsets of the first set is 112 more
than the number of subsets of the second set.Find the values of m and n.

Page 11 of 119
KVSRO/EKM/XI/MATHS

Long Answer questions (4 marks)

16.Let X = {1,2,3,4,…….10} A = {1,3,5,7,9} B = {1,2, 4 , 7,8}

Verify 1)(AUB)1 = A1 ∩ B1 2)(A ∩ B)1 = A1UB1

17.If X and Y are two sets such that n (X) =17 , n(Y) =23, n (X U Y) = 38,

find 1) n (X ∩ Y) 2) n ( X – Y ).

18. Let X = {1,2,3,……,8} A = {2,4,6,8,} B = {3,6,8}

Find 1) A1 2) (AUB)1 3) (A-B)1 4)(A ∩ B ) 1

19. If A = {1,2,3,4} , B = {3,4,5,6} C = {5,6,7,8}, D = {7,8,9,10 }

Find 1) AUBUC 2) AUBUD 3)A∩B∩C 4)A∩B∩D

20. Let A and B are sets.If A ∩ X =B∩X = ∅ and AUX = BUX, for some set X .Show that A = B

CASE STUDY QUESTIONS

Read the following passage and answer the questions given below(1 Mark for each question)

21.In a town of 10,000 families it was found that 40% family buy newspaper A ,20% family
buy the newspaper B ,10% buy newspaper C, 5% buy A and B ,3% buy B and C ,4% buy A and
C . If 2% buy all the three newspapers , find the number of families which buy
1) A only

a)14% b)33% c) 20%, d)10%

2) B only

a)14% b) 20% c)5% d)8%

3)Exactly one
a)48% b) 50% c)52% d)41%

4) Exactly two

a)6% b)5% c)4% d)3%

Read the following passage and answer the questions given below (1 Mark for each question)

22.There are 200 individuals with a skin disorder .120 had been exposed to chemical C 1,50 to
chemical C2 and 30 to both C1 and C2.Then number of individuals exposed to

1) chemical C1 but not chemical C2 is

a) 80 b)90 c) 100 d )110

2)chemical C2 but not chemical C1 is

a)20 b)30 c) 40 d) 50

Page 12 of 119
KVSRO/EKM/XI/MATHS

3) chemical C1 or chemical C2 is

a) 100 b) 110 c)130 d)140

4.None of the chemicals is


a)60 b)70 c)80 d)90

Read the following passage and answer the questions given below(1 Mark for each question)

23. In a survey of 600 students in a school, 150 students were found to be drinking Tea and

225 drinking Coffee ,100 were drinking both Tea and Coffee.Then the number students
were drinking

1)Tea only

a) 50 b)60 c)40 d) 30

2) Coffee only

a)50 b)75 c) 100 d) 125

3) Exactly one

a) 175 b)150 c)140 d)125

4) None of the drinks

a) 425 b)400 c)350 d)325

Read the following passage and answer the questions given below(1 Mark for each question)

24. In a class of 60 boys each playing atleast one game ,there are 45 boys who play cards and
30 boys play carrom ,then

Howmany boys play

1)Both games

a) 15 b) 20 c)25 d) 30

2) Cards only

a)10 b)20 c)30 d) 40

3) Carrom only

a)10 b)15 c)20 d)25

4) Exactly one game

a)40 b)45 c)50 d) 55

Page 13 of 119
KVSRO/EKM/XI/MATHS

ANSWERS

1)d 2) c 3) b 4) c 5) b
6) (A-B) = { 2,4,6,7,8,9,10}
1 7) n(A1∩ B1) = 300
𝑛
8) A = { x : x = ,n =1,2,3,4,5}
𝑛+3
11) X = { 3,5,9} 12) A – B = {1,2,3,4} B – A = {5}
13 ) A – B =A ∩ B1 = {3,6} 14) n(A) = 36 ,n(B) = 57

15) m=7,n = 4 16) ( A U B )1 = A1∩ B1 ={6,10}

17 ) n (X ∩ Y ) = 2, n(X – Y ) =15

19) 1) AUBUC = {1,2,3,4,5,6,7,8} 2)A U B U D = {1,2,3,4,5,6,7,8,9,10}

3) A ∩ B ∩ C = ∅ 4) A∩ B∩ D = ∅
21) 1) b 2)a 3)c 4)a

22) 1)b 2)a 3)d 4)a

23) 1)a 2)d 3)a 4)d

24) 1)a 2)c 3)b 4)b


SUMMARY OF THE CHAPTER

1. A well-defined collection of distinct objects is called a set.


2. A set which does not contain any element is called empty(null) set.
3. A set which consists of a definite number of elements is called a finite set.If a set contains
infinitely many elements is called an infinite set .
4. In roster form elements are listed out in the set.In set builder form all the elements are
represented by a single symbol(say x) along with the condition satisfied by x.
5. Two sets are said to be equal if they contain exactly the same elements.
6. The set B is said to be a subset of A if every element of B is present in A.
7. The set of subsets of A is called the power set of A .It is denoted by P(A).
8. The union of two sets A and B is the set of all elements which are either in A or in
B.Symbolically A 𝖴 𝐵= { x : x ∈ A or x ∈ B}.
9. The intersection of two sets A and B is the set of all elements which are common to A and
B. Symbolically A ∩ B ={ x : x ∈ A and x ∈ B }.If A ∩ B = ∅, A and B are called disjoint sets.
10. The difference of A and B in this order denoted by A – B is defined as the set of all
elements present in A ,but not in B.Symbolically A - B = {x : x ∈ A and x∉ B}
11. The compliment A denoted by A1is defined as the set of elements lying outside A and
inside the universal set X .
12. De – Morgan’s laws 1) (AUB)1 = A1 ∩ B1
13. For any twofinite sets A and B,n( AUB) =n(A)+n(B)- n(A∩B)
14. For any three finite sets n(AUBUC) =n(A)+n(B)+n(C) –n(A∩B)-n(B∩C)-n(C∩ A)+n(A∩B∩ C)

Page 14 of 119
KVSRO/EKM/XI/MATHS

RESULTS

1.AUA =A

2.AU ∅ =A

3.A∩A = A

4.A∩ ∅ = ∅

5.(A1)1 = A, X1 = ∅, ∅1 = X

6.AUA1 = X, A1 ∩ A = ∅

Page 15 of 119
KVSRO/EKM/XI/MATHS

Chapter 2
RELATIONS & FUNCTIONS
Syllabus

SUMMARY

Page 16 of 119
KVSRO/EKM/XI/MATHS

Identity function: Let R be the set of real numbers. Define the real valued function f : R → R by
y = f(x) = x for each x ∈ R. Such a function is called the identity function. Here the domain and
range of f are R.

Constant function Define the function f: R → R by y = f (x) = c, x ∈ R where c is a constant and


each x ∈ R. Here domain of f is R and its range is {c}The graph is a line parallel to x-axis.

Polynomial function A function f : R → R is said to be polynomial function if for each x in R, y =


f (x) = 𝑎𝑜+ 𝑎1 x + 𝑎2𝑥2+ ...+𝑎𝑛𝑥𝑛 , where n is a non-negative integer and 𝑎1 , 𝑎2 , a2 ,...𝑎𝑛∈R

Rational functions are functions of the type ( ) ( ) f x g x , where f(x) and g(x) are polynomial
functions of x defined in a domain, where g(x) ≠ 0.

The Modulus function The function f: R→R defined by f(x) = |x| for each x ∈R is called
modulus function.

Page 17 of 119
KVSRO/EKM/XI/MATHS

Signum function The function f:R→R defined by f(x) = -1 if x < 1

= 0 if x = 0

=1 if x > 1

is called the signum function. The domain of the signum function is R and the range is the set
{–1, 0, 1}.

Greatest integer function The function f: R → R defined by f(x) = [x], x ∈R assumes the value
of the greatest integer, less than or equal to x. Such a function is called the greatest integer
function.

Important formula :

 If n(A) = m, n(B) = n, then n(A × B) = mn and the total number of possible relations from
set A to set B = 2mn

Page 18 of 119
KVSRO/EKM/XI/MATHS

 If n(A) = m, n(B) = n, then n(A × B) = mn and the total number of possible nonempty
relations from set A to set B = 2𝑚𝑛 − 1
-

MCQ Questions

1) If A ={x,y,z} and B = {1,2} then number of non empty relations from A→ 𝐵 is

a) 6 b) 26c) 26 − 1 d) 25

2) Find the range of f(x) =𝑥2 + 2

a) [-2, 2] b) [2, ∞) c) [3, ∞) d) R

3) If A and B are 2 sets such that n(A× 𝐵) =6 and some of the pairs of

A× 𝐵are ={(3,2),(7,5),(8,5)] then which is true ?

a) A ={3,7,8} b) B={2,5,7} c)B ={3,5} d) A ={2,3,5,7,8}


4−𝑥
4) Domain and range of the function f(x) = is given by
𝑥−4

a) Domain=R, Range= {-1, 1} b) Domain=R-{1}, Range =R

c) Domain=R-{4}, Range= {-1} d) Domain=R-{-4}, Range= {-1, 1}

5) Let R be the relation on N defined by aRbiff a=𝑏𝑘 for some k∈ 𝑁.Which of the following
belongs to R?

a) (4,6) b)(1,7) c) (8,2) d) (2,32)

Page 19 of 119
KVSRO/EKM/XI/MATHS

6) The given below is a graph of a function. Name the graph,

a) Constant function b) Modulus function c) Identity function d) None of these

Short answer questions


𝑥 2 7 4
7) If ( + 1, 𝑦 − ) = ( , ) then find the value of x and y.
3 3 3 3

8) If A = {2, 5} then find A× 𝐴 × .

9) If A ={3,4,6},B={1,3} and C ={1,2,6} then find

i) A× (𝐵 ∩ 𝐶) ii) (𝐴 − 𝐵) × (𝐴 − 𝐶)

10) Let f be the subset of Z× 𝑍 × 𝑑𝑒𝑓𝑖𝑛𝑒𝑑 𝑏𝑦𝑓 = {(ab,a+b):a,b∈ 𝑍} Is f a function from Z to Z


? Justify your answer.

11) Draw the graph of greatest integer function,

12) If R = {(x, y): x and y are integers and𝑥2 + 𝑦2 = 64} is a relation then write R in roster
form.

Long answer questions

13) Let A = {1, 2}, B = {1, 2, 3, 4}, C = {5, 6} and D = {5, 6, 7, 8}. Verify that:
(i) A× (𝑩 ∩ 𝑪) = (A× 𝐵) ∩ (𝐴 × 𝐶)(ii) A× 𝑪 is a subset of B× 𝑫.

14) Let A = {1, 2, 3, 4, 6}. Let R be the relation on A defined by


{a,b):a,b∈ 𝑨, 𝒃𝒊𝒔𝒆𝒙𝒂𝒄𝒕𝒍𝒚𝒅𝒊𝒗𝒊𝒔𝒊𝒃𝒍𝒆𝒃𝒚𝒂}
(i) Write R in roster form.
(ii) Find the domain of R.
(iii) Find the range of R.

Page 20 of 119
KVSRO/EKM/XI/MATHS

15) Find the domain and range of the following real functions:
(i)f(x) = −|𝒙|
(ii) f(x) =√4 − 𝑥2

16) Find the domain and range of the real function defined by f(x) = √𝑥 − 2

𝑥2
17) Let f(x) = be a function from R into R. Determine the range of f.
𝑥2+1

18) Let f = {(1,12),(-1,-6),(2,21),(0,3)} be a function from Z to Z defined by f(x) = ax+b for


someintegers a and b.Find the value of a and b.

Case study questions:

19) Given 2 non empty sets A ={x:x< 4,x∈ 𝑁} 𝑎𝑛𝑑𝐵 = {𝑥: 𝑥 ≤ 2, 𝑥 ∈ 𝑊}


Based on the above information answer the following questions.
i) A× 𝐵 as the set of ordered pairs is
a) {(1,0),(1,2),(2,0),(2,1),(3,0),(3,1),(3,2)}
b) {(1,0),(1,1),(2,0),(2,1),(3,0),(3,1),(3,2)}
c) {(1,0),(1,1),1,2,(2,0),(2,1),2,2),(3,0),(3,1),(3,2)}
d) {(1,1),(2,2)}
ii) (A𝖴 𝐵) × (𝐴 ∩ 𝐵) as the set of ordered pairs is
a){(0,1),(0,2),(1,1),(1,2),(2,1),(2,2),(3,1),(3,2)}
b) ){(0,1),(0,2),(1,2),(2,1),(3,1),(3,2)}
c) {(0,1),(0,2),(1,1), 2,2),(3,1),(3,2)}
d) {(0,1),(0,2),(1,1),(1,2),(2,1),(2,2),(3,2)}
iii) (A−𝐵) × (𝐵 − 𝐴) as the set of ordered pairs is

a) {(3,1),(3,2)} b) {(2,0),(2,2)(3,1),(3,2)}
c){(0,1),(0,2),(1,1), 2,2),(3,1),(3,2)} d) {(3,0)}
iv)A relation R from A to B defined by R ={(x,y):x + y≥ 3, 𝑥 ∈ 𝐴, 𝑦 ∈ 𝐵}then R is
a) {(2,2),(3,1),(3,2)} b) {(1,0).(1,1),(1,2),(2,0),(2,1)(3,0)}
c){(1,2),((2,1),(2,2),((3,0),(3,1),(3,2)} d) {(2,2),(3,1),(3,2)}
v) Domain of R is

a) {1,2,3} b) {0,1,2} c) {1,2} d){0,1,2,3}

Page 21 of 119
KVSRO/EKM/XI/MATHS

20) Given 2 sets A ={2,3,4,5} and B={3,6,7,10} Based on the above given sets answer the
following questions.

i) R ={(a,b):a divides b, a∈ 𝐴, 𝑏 ∈ 𝐵} then R in roster form is

a) {(2,6),(2,10)(3,6),(4,10),(5,10)}
b) {(2,6),(2,10)(3,3),(5,10)}
c) {(2,6),(2,10)(3,3),(4,10),(5,10)}
d) {(2,6),(2,10)(3,3),(3,6),(5,10)}
ii) Co domain of R is

a) {2,3,4,5} b) {3,6,7,10} c) {3,6,10} d) none of these


iii) Range of R is
a) {2,3,4,5} b) {3,6,7,10} c) {3,6,10} d) none of these
iv) If 𝑅−1={(b,a): a divides b, a∈ 𝐴, 𝑏 ∈ 𝐵} then the set of ordered pairs in 𝑅−1 is

a) {(6,2),(10,2),(3,3),(6,3),(10,5)}
b) {(2,6),(2,10)(3,3),(3,6),(5,10)}
c) {(6,2),(10,2),(6,3),(10,5)}
d) none of these
v)Domain of 𝑅−1 is

a) {2,3,4,5} b) {3,6,7,10} c) {3,6,10} d) none of these

Answers:

1) c 2) b 3) a 4) c 5) c 6) b 7) x=4,y=2
8) A× 𝐴= {(2, 2) (2, 5), (5, 2), (5, 5)}
A× 𝐴 × 𝐴 = {(2,2,2),(2,5,2),(5,2,2),(5,5,2),(2,2,5),(2,5,5),(5,2,5),(5,5,5)}
9) B∩ 𝐶 = {1} and A× (𝐵 ∩ 𝐶 = {(3,1), (4,1), (6,1)}
A−𝐵 = {4,6} and A – 𝐶 = {3,4}𝑡ℎ𝑒𝑛(𝐴 − 𝐵) × (𝐴 − 𝐶)= {(4,3),(4,4),(6,3),(6,4)}
10) f(ab)= a+b then f(6) = f(1x6)=1+6=7 and f(6)=f(2x3)=2+3=5
Since 6 have 2 images so f is not a function.

Page 22 of 119
KVSRO/EKM/XI/MATHS

11)

12) R ={(0,8),(8,0),(0,-8),(-8,0)}

13)i) B∩ 𝐶 = ∅ and A× (𝑩 ∩ 𝑪) =∅
(A× 𝐵) = {(1,1),(1,2),(1,3),(1,4),(2,1),(2,2),(2,3),(2,4)} (𝐴 × 𝐶) = {(1,5)(1,6),(2,5),(2,6)
(𝐴 − 𝐵) × (𝐴 − 𝐶) = ∅
ii) (𝐴 × 𝐶) = {(1,5)(1,6),(2,5),(2,6) }
B× 𝐷 =
{(1,5),(1,6),(1,7)(1,8),(2,5),(2,6),(2,7),(2,8)(3,5)(3,6)(3,7)(3,8)(4,5)(4,6)(4,7)(4,8)}
Clearly A× 𝑪 is a subset of B× 𝑫.
14) R ={(1,1),(1,2)(1,3),(1,4)(1,6)(2,2),(2,4),(2,6),(3,3),(3,6),(4,4),(6,6)}
Domain ={1,2,3,4.6} and Range ={1,2,3,4.6}

15)i) Domain=R and Range =(−∞, 𝟎]

ii) Domain=[-2,2] Range =[0,2]

16) Domain=[2,∞) Range =[0,∞)

17) Range =[0,1)

18) a=9 and b = 3.

19) i)c ii)a iii)d iv) c v) a

20) i) d ii) c iii)c iv) a v) c

Page 23 of 119
KVSRO/EKM/XI/MATHS

Chapter 3
TRIGONOMETRIC FUNCTIONS
In this Chapter..............
 Measure of an Angle
 Trigonometric Function
 Trigonometric Identities
 Domain and range of trigonometric functions and their graphs
 Transformation Formulae

Measure of an Angle
There are two types of measure of angles :

Sexagesimal System (Degree Measure):One degree is divided into 60 equalparts, called minutes
and 1 minute is denoted by1′. One minute is divided into 60 equal parts, called second and 1second
is denoted by 1′′. Thus,° =60′ and 1′=60′′.

Circular System (Radian Measure)


A circle of radius r having an arc of length l will subtend an angle θ radian at the centre,

𝒍
𝑳𝒆𝒏𝒈𝒕𝒉 𝒐𝒇 𝒂𝒓𝒄
Where 𝜽 = = Or 𝒍 = 𝒓𝜽
𝒓𝑹𝒂𝒅𝒊𝒖𝒔

Relation between Degree and Radian

2π radian = 𝟑𝟔𝟎°orπ radian = 𝟏𝟖𝟎°


𝟏𝟖𝟎°
𝟏 𝒓𝒂𝒅𝒊𝒂𝒏 = = 𝟓𝟕°𝟏𝟔′(approximately)
𝝅
𝝅
𝟏° = 𝒓𝒂𝒅𝒊𝒂𝒏 = 𝟎. 𝟎𝟏𝟕𝟒𝟔 𝒓𝒂𝒅𝒊𝒂𝒏 (𝒂𝒑𝒑𝒓𝒐𝒙𝒊𝒎𝒂𝒕𝒆𝒍𝒚)
𝟏𝟖𝟎
Radian measure = 𝝅 × 𝑫𝒆𝒈𝒓𝒆𝒆 𝒎𝒆𝒂𝒔𝒖𝒓𝒆
𝟏𝟖𝟎
𝟏𝟖𝟎
Degree measure= × Radian measure
𝝅
Trigonometric Functions

In a right angled triangle, there are actually six possible trigonometric ratios or
functions. In right angled triangle ∆ABC

Page 24 of 119
KVSRO/EKM/XI/MATHS

𝐵𝐶 𝑃 𝐴𝐶 𝐻
sin 𝜃 = =
cosec 𝜃 = 𝐵𝐶 =
𝐴𝐶 𝑃
𝐻
𝐴𝐵 𝐴𝐶 𝐻 𝐵
cos 𝜃 = = sec 𝜃 = 𝐴𝐵 = 𝐵
𝐴𝐶 𝐻
𝐵𝐶 𝑃 𝐴𝐵 𝐵
tan 𝜃 = = cotθ = 𝐵𝐶 =
𝐴𝐵 𝐵 𝑃
Trigonometric Functions of Some Standard Angles

30 45 𝜋 270
60° ) 𝜋 180 360
Angle 𝜃° (𝜋) 𝜋
°( ) ( 90°( ) 135°( 3𝜋
°( )
6 4
3
2 °(𝜋) 2 (2𝜋)
1 1 √3 1
sin 𝜃 0 1 0 -1 0
2 √2 2 √2
√3 1 1 −1
cos 𝜃 1 0 -1 0 1
2 √2 2 √2
1 Not Not
tan 𝜃 0 1 √3 define -1 0 define 0
√3 d d

Trigonometric Functions in Terms of sine and cosine Functions


cosec x = 1 ,x ≠ nπ, where n is any integer
sin 𝑥

sec x= 1 . x≠ (2𝑛 + 1) ,where n is any integer


𝑐𝑜𝑠𝑥 2

cot x= 1 , x ≠n𝜋, where n is any integer


𝑡𝑎𝑛𝑥

Trigonometric Identities
sin2 𝜃 + cos2 𝜃 = 1
1 + tan2 𝜃 = sec2 𝜃
1 + cot2 𝜃 = cosec2 𝜃

Page 25 of 119
KVSRO/EKM/XI/MATHS

Signs of Trigonometric Functions in Various Quadrants

Quadrants
I All T- functions are positive
II Sin 𝜃 and Cosec 𝜃 are positive
All other ratios are negative
III Tan 𝜃 𝑎𝑛𝑑 Cot 𝜃 are positive
All other ratios are negative

IV Cos 𝜃 𝑎𝑛𝑑 Sec 𝜃 are positive


All other ratios are negative

I II III IV
All are positive sin 𝜃 is positive tan 𝜃 is positive 𝐜𝐨𝐬 𝜽 is positive

 Simple rule to remember


All Silver Tea Cups
After School To College

Trigonometric Ratios of Sum and Difference of Two Angles:


sin (𝑥 + 𝑦) = sin 𝑥cos 𝑦 + cos 𝑥sin 𝑦
sin(𝑥 − 𝑦) = sin 𝑥 cos 𝑦 − cos 𝑥 sin 𝑦
cos (𝑥 + 𝑦) = cos 𝑥cos 𝑦 − sin 𝑥sin 𝑦
cos(𝑥 − 𝑦) = cos 𝑥 cos 𝑦 + sin 𝑥 sin 𝑦
tan 𝑥+tan 𝑦 tan 𝑥−tan 𝑦
tan (𝑥 + 𝑦) = tan(𝑥 − 𝑦) =
1−tan 𝑥 tan 𝑦 1+tan 𝑥 tan 𝑦
cot 𝑥 cot 𝑦−1 cot 𝑥 cot 𝑦+1
cot (𝑥 + 𝑦) = cot(𝑥 − 𝑦) =
cot 𝑦+cot 𝑥 cot 𝑦−cot 𝑥

Transformation Formulae
Transformation of Product into Sum or Difference
 2 sin 𝑥 cos 𝑦 = sin(𝑥 + 𝑦) + sin(𝑥 − 𝑦)
 2cos 𝑥 sin 𝑦 = sin(𝑥 + 𝑦) − sin(𝑥 − 𝑦)
 2cos 𝑥 cos 𝑦 = cos(𝑥 + 𝑦) + cos(𝑥 − 𝑦)
 2sin 𝑥 sin 𝑦 = cos(𝑥 − 𝑦) − cos(𝑥 + 𝑦)

Page 26 of 119
KVSRO/EKM/XI/MATHS

Transformation of Sum or Difference into Product


𝑥+𝑦 𝑥−𝑦
 sin 𝑥 + sin 𝑦 = 2 sin sin
2 2
𝑥+𝑦 𝑥−𝑦
 sin 𝑥 − sin 𝑦 = 2 cos sin
2 2
𝑥+𝑦 𝑥−𝑦
 cos 𝑥 + cos 𝑦 = 2 cos cos
2 2
𝑥+𝑦 𝑥−𝑦
 cos 𝑥 − cos 𝑦 = −2 sin sin
2 2
Trigonometric Functions of Multiple and Sub-multiples of Angles:
𝜋
 sin 2𝑥 = 2sin 𝑥cos 𝑥 = 2 tan 𝑥 𝑥 ≠ 𝑛𝜋 + where n is an integer
1+tan2 𝑥 2
1−tan2 𝑥
 cos 2𝑥 = cos2 𝑥 − sin2 𝑥 = 2 cos2 𝑥 − 1 = 1 − 2 sin2 𝑥 =
1+tan2 𝑥
2 tan 𝑥
 tan 2𝑥 = 𝑖𝑓 2𝑥 ≠ 𝑛𝜋 + , where n is an integer
1−tan2 𝑥 2
 sin 3𝑥 = 3 sin 𝑥 − 4 sin3 𝑥
 cos 3𝑥 = 4 cos3 𝑥 − 3 cos 𝑥
3 tan 𝑥−tan3 𝑥 𝜋
 tan 3𝑥 = if 3𝑥 ≠ 𝑛𝜋 + , where n is an integer
1−3 tan2 𝑥 2

Trigonometric Functions of Sub-Multiple Angles


𝑥
2 tan2
 𝑥
sin 𝑥 = 2 sin 2 cos 2 = 1+tan2 2
𝑥
𝑥
𝑥 𝑥 𝑥 𝑥 1−tan2
cos 𝑥 = cos2 − sin2 = 2cos2 − 1 = 1 − 2 sin2 = 2
 2 2 2 2 1+tan2
𝑥

𝑥 2
2 tan
2
tan 𝑥 =
 1−tan2𝑥

2

𝑥 1+cos 𝑥
cos 2 = ±√
 2
 𝑥 1−cos 𝑥
sin = ±√
2 2

𝑥 1−cos 𝑥
tan 2 = ±√1+cos 𝑥

Trigonometric Ratios of Some Specific Angles


5−1
 𝐬𝐢𝐧 𝟏𝟖° = cos 72° = √
4
√10+2√5
 𝐜𝐨𝐬 𝟏𝟖° = sin 72° =
4
√𝟓+𝟏
 𝐜𝐨𝐬 𝟑𝟔° = 𝐬𝐢𝐧 𝟓𝟒° =
𝟒
√10−2√5
 𝐬𝐢𝐧 𝟑𝟔° = 𝐜𝐨𝐬 𝟓𝟒° =
4

Page 27 of 119
KVSRO/EKM/XI/MATHS

Domain and Rangeof Trigonometric functions


T- function Domain Range
sin 𝑥 R [-1,1]
cos 𝑥 R [-1,1]
𝜋
tan 𝑥 {x: 𝑥 ∈ 𝑅𝑎𝑛𝑑 𝑥 ≠ (2𝑛 + 1) , 𝑛 ∈ R
2
𝑍}
cot 𝑥 {x: x∈ 𝑅 𝑎𝑛𝑑 𝑥 ≠ 𝑛𝜋, 𝑛 ∈ 𝑍} R
cosec 𝑥 {x: x∈ 𝑅 𝑎𝑛𝑑 𝑥 ≠ 𝑛𝜋, 𝑛 ∈ 𝑍} {y: y∈ 𝑅, 𝑦 ≥ 1 𝑜𝑟 𝑦 ≤ −1}
𝜋
sec 𝑥 {x∈ 𝑅𝑎𝑛𝑑 𝑥 ≠ (2𝑛 + 1) , 𝑛 ∈ 𝑍} {y: y∈ 𝑅, 𝑦 ≥ 1 𝑜𝑟 𝑦 ≤ −1}
2

 For trigonometric functions of ( odd multiples of 𝜋 ± 𝜃) , the trigonometric


2
functions will interchange as sin 𝜃 ↔ cos 𝜃, sec 𝜃 ↔ cosec 𝜃, tan 𝜃 ↔ cot 𝜃
3𝜋
Example: sin( + 𝜃) = − cos 𝜃 [In fourth quadrant sin −𝑣𝑒]
2

 For trigonometric functions of (multiples of 𝜋 ± 𝜃), there is no change in the


trigonometric functions
Example: tan(𝜋 + 𝜃) = tan 𝜃[ In third quadrant tan +𝑣𝑒]
 sin(−𝜃) = − sin 𝜃 ,cos(−𝜃) = cos 𝜃

MULTIPLE CHOICES QUESTIONS ( MCQs) - [1 mark]


(Choose the correct answer out of the given four options)

1. If α + β = π/4, then the value of (1 + tan α) (1 + tan β) is


(a) 1(b) 2(c) -2(d) Not defined
2. The value of tan 1° tan 2° tan 3° … tan 89° is
(a) 0 (b) 1 (c) ½ (d) Not defined
3.The value of sin(45 + 𝜃) − cos(45 − 𝜃) is
(a) 2 cos (b) 2 sin 𝜃 (c) 1 (d) 0
4. If sin 𝜃 + cos 𝜃 = 1,then the value of sin 2𝜃 is equal to
(a) 1 (b) 1 (c) 0 (d) −1
2

5. If A lies in second quadrant and 3 tan 𝐴 + 4 = 0, then the value of 2 cot 𝐴 − 5 cos 𝐴 + sin 𝐴 is
equal to
−53
(a) 23 (b) (c) 37 (d) 7
10 10 10 10

Page 28 of 119
KVSRO/EKM/XI/MATHS

ANSWERS
1. (b) 2 2. (b) 1 3. (d) 0 4. (c) 0 5. (a)23
10

SHORT ANSWER TYPE QUESTIONS (2 MARKS)


2. Find the radius of the circle in which a central angle of 60° intercepts an arc of
𝜋
length37.4cm (use π = 22/7). 𝑙=r , where 𝜃= 60° = 𝑙=37.4 cm,
3
l
Radius =
θ

Ans:- 35.7 cm
3. Show that: tan 3x tan 2x tan x = tan 3x – tan 2x – tan x.
13𝜋
4. Find the value of tan
12
13𝜋 𝜋 𝜋
Ans :-tan = tan( 𝜋 + ) = tan
12 12 12
𝜋 𝜋
= tan( − ) =2−√3
4 6
0
5. Evaluate tan 75 .
Ans:-2 + √3
6. Prove that Sin (40 + θ). Cos (10 + θ) – Cos (40 + θ). Sin (10 + θ) = 1
2
Ans:-L. H. S = Sin (40 + θ). Cos (10 + θ) – Cos (40 + θ). Sin (10 +θ)
= sin( 40 + 𝜃 − 10 − 𝜃)
= sin 30 =1 RHS
2

7. Prove that tan 560 = 𝐜𝐨𝐬 𝟏𝟏°+𝐬𝐢𝐧 𝟏𝟏°


𝐜𝐨𝐬 𝟏𝟏°−𝐬𝐢𝐧 𝟏𝟏°
tan 45°+tan 11°
LHS = tan( 45° + 11°) = 1−tan 45° tan 11°
1+tan 11° cos 11°+sin 11°
= = = RHS
1−tan 11° cos 11°−sin 11°

𝐭𝐚𝐧 𝟔𝟗+𝐭𝐚𝐧 𝟔𝟔
8. Prove that 𝟏−𝐭𝐚𝐧 𝟔𝟗 𝐭𝐚𝐧 𝟔𝟔 = −𝟏
LHS = tan( 69 + 66) = tan 135
=tan(180 − 45)
= − tan 45 = -1 = RHS

Page 29 of 119
KVSRO/EKM/XI/MATHS

9. The minute hand of a watch is 1.5 cm long. How far does it tip move in 40
minute?

ANS :-Angle made in 1 min = = 600


4𝜋radian =
Angle made in 40 mint= 240° = 𝜃
3
Radius = 1,5 cm
𝑙 = 𝑟𝜃 = 2𝜋𝑐𝑚= 6.28cm
𝑝+𝑞
10.If tan(𝐴 + 𝐵) =𝑝, tan(𝐴 − 𝐵) = 𝑞, then show that tan 2𝐴 =
1−𝑝𝑞
2A = (A+B) +(A-B)
tan( 𝐴+𝐵)+tan(𝐴−𝐵) 𝑝+𝑞
tan 2𝐴 = =
1−tan(𝐴+𝐵) tan(𝐴−𝐵) 1−𝑝𝑞

𝜋 5𝜋 𝜋
11. Show that cot2 + cosec + 3 tan2 = 6
6 6 6
−3 3𝜋
12. Find the values of cos 𝜃 and tan 𝖯 if sin 𝜃 = and 𝜋 < 𝜃 <
5 2
−4 3
Ans:-cos 𝜃 = , tan 𝜃 =
5 4

13. Prove that sin 10° + sin 20° + sin 40° + sin 50° = sin 70° + sin 80°
Ans:-LHS = (sin 50° + sin 10°) + (sin 20° + sin 40°)
= 2 sin 30° cos 20° + 2 sin 30° cos 10°
= cos 20° + cos 10°
=cos(90° − 70°) + cos(90° − 80°)
=sin 70° + sin 80° RHS
14.Prove that cos 𝐴 + cos(120 − 𝐴) + cos(120 + 𝐴) = 0
240°
LHS = cos 𝐴 + 2 cos cos(−𝐴)
2
−1
= cos 𝐴 + 2 × × cos 𝐴 = 0 RHS
2
1 3𝜋
15.Find the value of tan(𝛼 + 𝛽), given that cot 𝛼 = , 𝛼 ∈ (𝜋, )
2 2
−5 𝜋
and sec 𝛽 = , 𝛽 ∈ ( 2 , 𝜋)
3
2
Ans:-tan(𝛼 + 𝛽) =
11

Page 30 of 119
KVSRO/EKM/XI/MATHS

16.A horse is tied to a post by a rope. If the horse moves along a circular path
always keeping therope tight and describes 88m when it has traced out 72° at
the centre. Find the length of the rope.
Ans:- AB = 88m, PA = r metres, 𝜃 = 72° = 2𝜋radian.
5
𝑙 5
𝑟= = 88× = 70 metres
𝜃 2𝜋

LONG ANSWER TYPE QUESTIONS


𝑏 𝑎+𝑏 𝑎−𝑏
17. If tan 𝑥 = , then find the value of √ +√
𝑎 𝑎−𝑏 𝑎+𝑏
𝑎+𝑏 𝑎−𝑏 2𝑎
Ans:-√ +√ =
𝑎−𝑏 𝑎+𝑏 √𝑎2−𝑏2

1 2𝑎 2 2 cos 𝑥
=𝑎 × 𝑏2
= √1−tan2 =
√1− 𝑥 √cos 2𝑥
𝑎2

𝜋 3
18. Prove that cos2 𝑥 + cos2 (𝑥 + 𝜋) + cos2 (𝑥 − 3) =2
3
−1
19. If sin 𝑥 = , x lies in quadrant IV then find the values of :
2
𝑥 𝑥 𝑥
sin , cos and tan
2 2 2

𝑥 𝑥
√3, sin = √2−√3 𝑥 √2+√3 2−√3
Ans :-cos 𝑥 = , cos = − , tan = −√ 2+√3
2 2 2 2 2 2

20. sin 𝐴+sin 3𝐴+sin 5𝐴+sin 7𝐴 = tan 4𝐴


Prove that
cos 𝐴+cos 3𝐴+cos 5𝐴+cos 7𝐴

(sin 7𝐴+sin 𝐴)+(sin 5𝐴+sin 3𝐴)


Hint :
(cos 7𝐴+cos 𝐴)+(cos 5𝐴+cos 3𝐴)

4 tan 𝑥( 1−tan2 𝑥)
21. Prove that tan 4𝑥 =
1−6 tan2 𝑥+tan4 𝑥
Hint:- tan 4𝑥 = tan 2(2𝑥)
2 tan
Put 2𝑥 = A, tan 4𝑥 = tan 2𝐴 = ,replace A by 2𝑥
1−tan2 𝐴

22. Prove that :


𝛼+𝛽 𝛽+𝛾 𝛾+𝛼
cos 𝛼 + cos 𝛽 + cos 𝛾 + cos(𝛼 + 𝛽 + 𝛾) = 4 cos cos cos
2 2 2
Ans:- (cos 𝛼 + cos 𝛽) + {cos(𝛼 + 𝛽 + 𝛾) + cos 𝛾}
𝛼+𝛽 𝛼−𝛽 𝛼+𝛽+2𝛿 𝛼+𝛽
= 2 cos cos + 2 cos cos
2 2 2 2
𝛼+𝛽 𝛼−𝛽 𝛼+𝛽+2𝛾
=2 cos [cos + cos ] (Proceed further)
2 2 2

Page 31 of 119
KVSRO/EKM/XI/MATHS

CASE STUDY QUESTIONS


1

ANSWER © 4.19
2.

Answer.(b) 22:13

Page 32 of 119
KVSRO/EKM/XI/MATHS

Chapter 4
COMPLEX NUMBERS
Gist of the Lesson

 Complex Number:
A number of the form x + iy where x and y are real numbers and i=√−1 is called a
complex number. x is called real part and y is called imaginary part
 Purely real and purely imaginary complex number:
A complex number x+iy is purely real if its imaginary part is zero(y=0) and purely
imaginary if its real part is zero(x=0)
 Equality of two complex numbers:
Two complex numbers Z1= a+iband Z2= c+id is equal iff a=c and b=d
 Algebra of complex numbers:
Addition Sum of two complex numbers Z1= a+ib and Z2 = c+id is a complex number and
is z=(a+c)+i(b+d)
 Properties of addition:
Commutative: Z1+Z2= Z2+Z1
Associative: Z1+(Z2+Z3)=(Z1+Z2)+Z3
Identity exists: (For any complex number Z, Z+0=0+Z =Z i.e., the complex number 0=
0+i0 is the identity element for addition)
Inverse exists: (For any complex number Z= a+ib,there exists -Z=-a+-ib such that Z+-
Z=0=-Z+Z,-Z is called the additive inverse of Z)
 Subtraction:
Let Z1and Z2 are two complex numbers,then Z1-Z2= Z1+-Z2

 Multiplication:
Let Z1=a+ib and Z2=c+id be two complex numbers,then
Z1.Z2= (a+ib)(c+id) =(ac – bd)+i(ad+bc)
i.e.,Z1Z2=[Re(Z1)Re(Z2)-Im(Z1)Im(Z2)]+i[Re(Z1)Im(Z2)+Re((Z2)Im(Z1)]

 Properties of multiplication:
Commutative: Z1Z2=Z2Z1
Associative: (Z1Z2)Z3 =Z1(Z2Z3)
Identity exists: (For a complex number Z, there exists the complex number
1= 1+i0 such that Z.1=1.Z=Z, the number 1+i0 is the identity element for multiplication.

 Existence of multiplicative inverse:


Corresponding to every non- zero complex numberZ=a+ib,there exists a complex
numberZ1=x+iy such that Z.Z1=1=Z1Z

Page 33 of 119
KVSRO/EKM/XI/MATHS

 Division
The division of a complex number Z1 by a non- zero complex numbersZ2 is defined as the
multiplication of Z1 by the multiplicative inverse of Z2 and is denoted by Z1/Z2
 Conjugate of a complex number
Let Z=a+ib be a complex number, then the conjugate of Z is denoted by Z̅and is Z̅= a-ib

 Properties of conjugate
If Z, Z1, Z2 are complex numbers, then

i.̅Z
(̅) = Z
ii. Z + Z̅= 2 Re(Z)
iii. Z − Z̅= 2 Im(Z)
iv. Z = Z̅⇒ Z is purely real
v. Z + Z̅= 0 ⇒ Z is purely imaginary
vi. ZZ̅= [Re(Z)]2 + [Im(Z)]2
vii. ̅Z̅1+ ̅ ̅Z̅2= ̅Z̅1+ ̅Z̅2
viii. ̅Z̅1− ̅ ̅Z̅2= ̅Z̅1− ̅Z̅2
ix. Z̅ 1̅Z̅2= ̅Z̅Z 1̅2
̅ Z1̅ ̅ ̅Z̅1
x.
( Z2) = ̅Z̅ ⋅2 , Z2 ≠ 0.
 Modulus of a complex number
The modulus of a complex number Z=a+ib is denoted by |z| and is defined as

|Z|=√a2 + b2 = √Re(z2) + Im(z2)


 Properties of modulus
|Z|=0 iff Z=0
|Z|=|Z̅|=|-Z|
ZZ̅=|Z|2
|Z1Z2|=|Z1||Z2|
Z1 |Z1|
| |= , provided |Z | ≠ 0
Z2 |Z2| 2

 Reciprocal of a complex number


Let Z= a+ib be a non- zero complex number,then
1 1 1 a − ib a − ib Z̅
= = × = =
Z a + ib a + ib a − ib a2 + b2 |Z|2

 Argand plane
The plane having a complex number assigned to each of its point is called the complex
plane or the Argand plane
The complex number a+ib corresponds to the ordered pair P (a,b) and can be represented
geometrically as the unique point P (a,b) in the XY plane

Page 34 of 119
KVSRO/EKM/XI/MATHS

Multiple Choice Questions

1. The value of√−25 + 3√−4 + 2√−9 is


(a) 13i (b) -13i (c) 17in (d) -17i
1+i
2. The least value of n for which ( ) is real, is
1−i
(a) 1 (b) 2 (c) 3 (d) 4
3. If a + ib = c + id, then
(a) a2 + c2 = 0 (b) b2 + c2 = 0 (c) b2 + d2 = 0 (d) a2 + b2 = c2 + d2
(𝟏−𝐢)𝟑
4. The simplified value of is
(𝟏−𝐢𝟑)
a) -1 b)-2 c)-I d)2i

5. If z = 2 -3i, then value of z2 – 4z + 13 is


a) 1 b)-1 c)0 d)None of these

Short Answer Type Questions


6. Find the modulus i25 + (1 + 3i)3
1+i 1−i
7. Find the modulus of ( ) − ( )
1−i 1+i
8. Express in the form of a + ib. (1+3i)-1
9. If (1 + i)z = (1 − 𝑖)z̅, then show that z = −iz̅
c+i b 2c
10. If a + ib = , where a, b are real prove that a2 + b2 = 1 and =
c−i a c2−1

Long Answer Type Question


z1+z 2+1
11. If z1 = 2-i and Z2 = 1+i Find | |
z1−z2+i
12. For any two complex numbers Z1 and Z2, show that
Re(Z1Z2) = ReZ1ReZ2– ImZ1ImZ2
13. If |z + 1| = z + 2(1 + i), then find the value of z.

Answers
1. 17 i
2. 2
3. a2 + b2 = c2 + d2.
4. -2
5. 0
Solutions
6. 𝐢𝟐𝟓 + (𝟏 + 𝟑𝐢) = (𝐢𝟒)𝟔𝐢 + 𝟏 + 𝟐𝟕𝐢𝟑 + 𝟑(𝟏)(𝟑𝐢)(𝟏 + 𝟑𝐢)
= 𝐢 + (𝟏 − 𝟐𝟕𝐢 + 𝟗𝐢 + 𝟐𝟕𝐢𝟐)
= 𝐢 + 𝟏 − 𝟏𝟖𝐢 − 𝟐𝟕

Page 35 of 119
KVSRO/EKM/XI/MATHS

= −𝟐𝟔 − 𝟏𝟕𝐢

= |−𝟐𝟔 − 𝟏𝟕𝐢| = √(−𝟐𝟔)𝟐 + (𝟏𝟕)𝟐


= √𝟔𝟕𝟔 + 𝟐𝟖𝟗
= √𝟗𝟔𝟓

1+i 1−i
7. Given: ( )−( )
1−i 1+i
Simplify the given expression, we get:
1+i 1−i (1+i)2−(1−i)2 1+i2+2i−1−i2+2i
( ) − ( )= ( )( ) = 2 2
1−i 1+i 1+i 1−i 1 −i
1 + i2 + 2i − 1 − i2 + 2i
=
12 − i2
Now, cancel out the terms,
4i
=2
=2i
Now, take the modulus,
1+i 1−i 2
|( ) − ( )|=|2i| = √2 = 2
1−i 1+i
1+i 1−i
Therefore, the modulus of ( )−( ) is 2.
1−i 1+i

1
8. (1 + 3i)−1 = ×
1−3i
=
1−3i
1+3i 1−3i 12−(3i)2
1−3i 1−3i 1−3i 1 3i
= = = = −
12−(3i)2 1+9 10 10 10

1−i
9. z = z̅
1+i
(1−i)(1−i) 1−2i+i2 1−2i−1
= (1+i)(1−i)z̅= z̅= z̅ = −iz̅
1+1 2

c+i
10. a + ib = (given)
c−i

c2 + 2ci + 𝑖2
a + ib =
c2 − i 2
c2 − 1 2c
a + ib = c2 + 1 + c2 + 1 i
c2 − 1 2c
a= 2 , b= 2
c +1 2c +1 2
c2 − 1 4c
a 2 + b2 = ( 2 ) +
c +1 (c2+1)2
(c2 + 1) 2
=1
(c2 + 1)2
2c
b
= c 2 +1 =b = 2c
a c2−1 a c2−1
c2 +1

Page 36 of 119
KVSRO/EKM/XI/MATHS

11. Ans. z1 + z2 + 1 = 2 – i + 1+ i + 1 = 4
z1 - z2 + i = 2 – i + 1- i + i = 1 – i

z +z +1 4 |4| |4| 4 4 2 4√2


| z1−z 2+i | = | |=| |= 2 2 = = x√ = = 2√2
1 2 1−i 1−i √1 +(−1) √2 √2 √2 √2

12. Given: z1 and z2 are the two complex numbers


To prove: Re(z1z2) = Rez1 Rez2– Imz1Imz2
Let z1 = x1+iy1 and z2 = x2+iy2
Now, Z1 Z2 =(x1+iy1)(x2+iy2)
Now, split the real part and the imaginary part from the above equation:
⇒x1(x2+iy2) +iy1(x2+iy2)
Now, multiply the terms:
= x1x2+ix1y2+ix2y1+i2y1y2
We know that, i2 = -1, then we get
= x1x2+ix1y2+ix2y1-y1y2
Now, again separate the real and the imaginary part:
= (x1x2 -y1y2) +i (x1y2+x2y1)
From the above equation, take only the real part:
⇒ Re (Z1Z2) = (x1x2 -y1y2)
It means that,
⇒Re(Z1Z2) = ReZ1 ReZ2– ImZ1ImZ2
Hence, the given statement is proved.

13. Ans: We have |z + 11 = z + 2(1+ i)


Putting z = x + iy, we get
Then, |x + iy + 11 = x + iy + 2(1 + i)
⟹|x + iy + 1|=x + iy + 2(1 +i)
Comparing the real and imaginary parts, we get
√(𝐱 + 𝟏)𝟐 + 𝐲𝟐 = 𝐱 + 𝟐
𝐲 + 𝟐 = 𝟎 ⇒ 𝐲 = −𝟐
Putting y = −2 into √(𝐱 + 𝟏)𝟐 + 𝐲𝟐 = 𝐱 + 𝟐 we get
(𝐱 + 𝟏)𝟐 + (−𝟐)𝟐 = (𝐱 + 𝟐)𝟐
⇒ 𝐱𝟐 + 𝟐𝐱 + 𝟏 + 𝟒 = 𝐱𝟐 + 𝟒𝐱 + 𝟒
1
⇒ 2x = 1 ⇒ x =
2
1
z = x + iy = − 2i
2

Page 37 of 119
KVSRO/EKM/XI/MATHS

Case Study Question

14. Consider two complex numbers Z1= 1 + i, Z2 = 2 – i, based on the information answer the
following questions
6 3
1) The value of 𝑍1 + 𝑍1 =
a) –4i – 10 b)–2 – 10i c)2 – 10i d)–2 + 10i

2) The value of 𝑍1−1


a) (1 – i)/2 b)(1 + i)/2 c)(3 – i)/2 d)(3 + i)/2

Z2
3) Conjugate of is
Z1
3 5ⅈ 1 3ⅈ 3 5ⅈ
a) 1 3ⅈ b) + c) + d) −

2 2 2 2 2 2 2 2

4) Conjugate of 𝑍1 ⋅ 𝑍̅2
a) 1 – 3i b)2 – 3i c)2 + 3i d)1 + 3i
𝑍1
5) Which quadrant does lie
𝑍2
a) 1st b)2nd c)3rd d)4th

Answers
1) b)
2) a)
3) c)
4) a)
5) a)

Page 38 of 119
KVSRO/EKM/XI/MATHS

Chapter 5
LINEAR INEQUALITIES
MULTIPLE CHOICE QUESTIONS

1) The solution set of the inequality 4𝑥 + 3 < 6𝑥 + 7is


(A) [−2, ∞] (B) (−∞, −2) (C) (−2, ∞) (D) none of these
2) If 3𝑥 + 8 > 2, then which of the following is true?
(A) 𝑥 ∈ { −1, 0, 1, 2, … … . }, when 𝑥 is an integer (B) 𝑥 ∈ [−2, ∞), when 𝑥 is a real number
(C) Bothe (A) and (B) (D) none of the above
𝑥+3
3) The solution set of the inequality ≥ 1is
𝑥+4

(A) (−∞, −4) (B) (−∞, 4) (C) (−4, ∞) (D) (4, ∞)


1
4) The solution set of the inequality |3𝑥 − 2| ≤ is
2
1 3 1 3 1 5 1 5
(A) [ , ] (B) [ , ] (C) [ , ] (D) [ , ]
2 2 2 4 2 3 2 6
3𝑥−4 𝑥+1
5) The graph of the solutions of inequality ≥ − 1 on number line is
2 4

(A)

(B)

(C )

(D)
SHORT ANSWER QUESTIONS (2 marks)
6) Solve 1 − 3𝑥 > 10 and represent the solution graphically on a number line
𝑥+8
7) Solve >1
𝑥+2

8) Solve −3 ≤ 4 − 7𝑥 < 18
𝑥−5
9) Solve <0
𝑥+2

10) Solve |𝑥 − 2| ≥ 6
11) Solve the system of inequalities 𝑥 + 2 > 0and 3𝑥 − 8 < 1.

Page 39 of 119
KVSRO/EKM/XI/MATHS

12) Solve |𝑥 − 1| ≤ 2
13) The sum of three consecutive integers must not be more than 12. What are the integers?
14) Solve |𝑥| < 4 and represent the solution set on the number line.

15) Anu obtained 73, 67 and 72 marks in mathematics test . How many should she get in his fourth
test, so as

to have an average of at least 75 .


LONG ANSWER QUESTIONS

16) The cost and revenue functions of a product are given by 𝐶(𝑥) = 20𝑥 + 4000 and 𝑅(𝑥) =
60𝑥 + 2000
respectively , where 𝑥 is the number of items produced and sold. How many items must be sold to
realise

some profit?

17) A solution is to be kept between 40℃ 𝑎𝑛𝑑 50℃.What is the range of temperature in degree
Fahrenheit,

the if the conversion formula is 𝐹 = 9 𝐶 + 32.


5

18) A solution of 9% acid is to be diluted by adding 3% acid solution to it. The resulting mixture is
to be more
than 5% but less than 7% acid. If there is 460 litres of the 9% solution, How many litres of 3%
solution
will have to be added?
CASE STUDY QUESTIONS
19) In drilling world’s deepest hole, the Kola Superdeep Borehole, the deepest manmade hole on
Earth and
deepest artificial point on Earth, as a result of a scientific drilling project, it was found that the
temperature
T in degree Celsius, 𝑥 km below the surface of Earth, was given by: = 30 + 25 (𝑥 – 3), 3 <
𝑥 < 15.
If the required temperature lies between 200o C and 300o C, then

Page 40 of 119
KVSRO/EKM/XI/MATHS

1. the depth, 𝑥 will lie between


1. 9 km and 13 km
2. 9.8 km and 13.8 km
3. 9.5 km and 13.5 km
4. 10 km and 14 km

2. If |𝑥| <5 then the value of 𝑥 lies in the interval


1. (−∞, −5)
2. (∞, 5)
3. (−5, ∞)
4. (−5, 5)

3. Graph the following inequality on the number line: x > −32

1.

2.

3.
4.

20) Sheena is a Psychology student and now a days she is learning about IQ (Intelligence Quotient
) of a
𝑀𝐴
person. . She knows the result ,𝑄 = × 100, where MA stands for Mental Age and CA stands
𝐶𝐴
for Chronological Age.

Page 41 of 119
KVSRO/EKM/XI/MATHS

1) What would be the range of mental age if a group of children with chronological age of 15 years have
the IQ range as 90 ≤ 𝐼𝑄 ≤ 150?
2) What would be the range of IQ if a group of children with chronological age of 12 years have the
mental age range as 9 ≤ 𝑀𝐴 ≤ 15.
𝑨𝑵𝑺𝑾𝑬𝑹𝑺

1) 4𝑥 + 3 < 6𝑥 + 7 ⇒ 4𝑥 − 6𝑥 < 7 − 3

⇒ −2𝑥 < 4
4
⇒𝑥>
−2
⇒ 𝑥 > −2
Answer: (C)(−2, ∞)
2)3𝑥 + 8 > 2 ⇒ 3𝑥 > 2 − 8

⇒ 3𝑥 > −6
−6
⇒𝑥>
3
⇒ 𝑥 > −2
Answer: (A) 𝑥 ∈ { −1, 0, 1, 2, … … . }, when 𝑥 is an integer

𝑥+3 𝑥+3
3) ≥1⇒ −1 ≥0
𝑥+4 𝑥+4

𝑥 + 3 − (𝑥 + 4)
⇒ ≥0
𝑥+4
−1
⇒ ≥0 ⇒𝑥+4 <0
𝑥+4
⇒ 𝑥 < −4
Answer: (A) (−∞, −4)
1 −1 1
4) |3𝑥 − 2| ≤ 2 ⇒ 2
≤ 3𝑥 − 2 ≤ 2

−1 1
⇒ + 2 ≤ 3𝑥 ≤ +2
2 2
3 5
⇒ ≤ 3𝑥 ≤
2 2
1 5
⇒ ≤𝑥≤
2 6
1 5
Answer: (D) [ , ]
2 6

Page 42 of 119
KVSRO/EKM/XI/MATHS

3𝑥−4
≥ 𝑥+1 − 1 ⇒ 4 × 3𝑥−4 𝑥+1
5) ≥4× −4×1
2 4 2 4

⇒ 2(3𝑥 − 4) ≥ 𝑥 + 1 − 4
⇒ 6𝑥 − 8 ≥ 𝑥 − 3
⇒ 6𝑥 − 𝑥 ≥ −3 + 8
⇒ 5𝑥 ≥ 5
⇒𝑥≥1

Answer: (A)
SHORT ANSWER QUESTIONS
6) 1 − 3𝑥 > 10 ⇒ −3𝑥 > 10 − 1

⇒ −3𝑥 > 9
⇒ 𝑥 < −3

𝑥+8 𝑥+8
7) >1⇒ −1 >0
𝑥+2 𝑥+2

𝑥 + 8 − (𝑥 + 2)
⇒ >0
𝑥+2
6
⇒ >0
𝑥+2
⇒𝑥+2 >0
⇒ 𝑥 > −2. The solution is (−2, ∞)
8) −3 ≤ 4 − 7𝑥 < 18 ⇒ −3 − 4 ≤ 4 − 7𝑥 − 4 < 18 − 4
⇒ −7 ≤ −7𝑥 < 14
−7 −7𝑥 14
⇒ ≥ >
−7 −7 −7
⇒ 1 ≥ 𝑥 > −2or−2 < 𝑥 ≤ 1 = (−2,1]
𝑥−5
9) < 0 ⇒ 𝑥 − 5 > 0 𝑎𝑛𝑑 𝑥 + 2 < 0 𝑂𝑅 𝑥 − 5 < 0 𝑎𝑛𝑑 𝑥 + 2 > 0
𝑥+2

𝑥 − 5 > 0 𝑎𝑛𝑑 𝑥 + 2 < 0 ⇒ 𝑥 > 5 𝑎𝑛𝑑 𝑥 < −2which is not possible

𝑥 − 5 < 0 𝑎𝑛𝑑 𝑥 + 2 > 0 ⇒ 𝑥 < 5 𝑎𝑛𝑑 𝑥 > −2. So the solution is (−2, 5)
10) |𝑥 − 2| ≥ 6 ⇒ −6 ≥ 𝑥 − 2 ≥ 6

Page 43 of 119
KVSRO/EKM/XI/MATHS

⇒ −6 + 2 ≥ 𝑥 ≥ 6 + 2
⇒ −4 ≥ 𝑥 ≥ 8 . So the solution is 𝑥 ≤ −4 𝑜𝑟 𝑥 ≥ 8
11) 𝑥 + 2 > 0and3𝑥 − 8 < 1 ⇒ 𝑥 > −2 𝑎𝑛𝑑 3𝑥 < 1 + 8
⇒ 𝑥 > −2 𝑎𝑛𝑑 3𝑥 < 9
⇒ 𝑥 > −2 𝑎𝑛𝑑 𝑥 < 3
So the solution is −2 < 𝑥 < 3 OR (−2, 3)
12) |𝑥 − 1| ≤ 2 ⇒ −2 ≤ 𝑥 − 1 ≤ 2

⇒ −2 + 1 ≤ 𝑥 ≤ 2 + 1
⇒ −1 ≤ 𝑥 ≤ 3 OR [−1, 3]
13) Let the consecutive integers be 𝑥, 𝑥 + 1 and 𝑥 + 2

Given, 𝑥 + 𝑥 + 1 + 𝑥 + 2 ≤ 12
3𝑥 + 3 ≤ 12
3𝑥 ≤ 12 − 3 = 9
𝑥≤3
14) |𝑥| < 4 ⇒ −4 < 𝑥 < 4

15) Let the marks in the fourth test be 𝑥


73+67+72+𝑥
Given ≥ 75
4

212 + 𝑥 ≥ 75 × 4
212 + 𝑥 ≥ 300
𝑥 ≥ 300 − 212 = 88
The mark in the fourth test should be at least 88
16) For getting profit revenue should be more than the cost

𝑅(𝑥) > 𝐶(𝑥)


60𝑥 + 2000 > 20𝑥 + 4000
60𝑥 − 20𝑥 > 4000 − 2000
40𝑥 > 2000
2000
𝑥> ⇒ 𝑥 > 50. The manufacturer must sell more than 50 items to realise the profit.
40

Page 44 of 119
KVSRO/EKM/XI/MATHS

17) 40 < 𝐶 < 45 ⇒ 9 × 40 < 9 × 𝐶 < 9 × 45


5 5 5
9 9 9
⇒ × 40 + 32 < × 𝐶 + 32 < × 45 + 32
5 5 5
⇒ 72 + 32 < 𝐹 < 81 + 32
⇒ 104 < 𝐹 < 113
𝐻𝑒𝑛𝑐𝑒 𝑡ℎ𝑒 𝑟𝑒𝑞𝑢𝑖𝑟𝑒𝑑 𝑟𝑎𝑛𝑔𝑒 𝑜𝑓 𝑡𝑒𝑚𝑝𝑒𝑟𝑎𝑡𝑢𝑟𝑒 𝑖𝑠 𝑏𝑒𝑡𝑤𝑒𝑒𝑛 104℉ 𝑎𝑛𝑑 113℉
18) Let 𝑥 be the number of litres of 3% acid solution. Then total mixture = (460 + 𝑥)𝑖𝑡𝑟𝑒𝑠

Given, 5% of (460 + 𝑥) < 3% 𝑜𝑓 𝑥 + 9% 𝑜𝑓 460 <7% of (460 + 𝑥)


5 3 9 7
(460 + 𝑥) < ×𝑥+ × 460 < (460 + 𝑥)
100 100 100 100
5(460 + 𝑥) < 3𝑥 + 4140 < 7(460 + 𝑥)

2300 + 5𝑥 < 3𝑥 + 4140 < 3220 + 7𝑥


2300 + 5𝑥 < 3𝑥 + 4140, 3𝑥 + 4140 < 3220 + 7𝑥

5𝑥 − 3𝑥 < 4140 − 2300 , 4140 − 3220 < 7𝑥 − 3𝑥


2𝑥 < 1840 , 920 < 4𝑥

𝑥 < 920 , 230 < 𝑥 ⇒ 230 < 𝑥 < 920


Number of litres of 3% solution is more than 230 litres and less than 920 litres

CASE STUDY QUESTIONS


19) (1) 200 < 30 + 25 (𝑥 – 3) < 300

200 < 30 + 25𝑥 − 75 < 300


200 < 25𝑥 − 45 < 300
200 + 45 < 25𝑥 < 300 + 45
245 < 25𝑥 < 345
9.8 < 𝑥 < 13.8
Answer: 2) 9.8 < 𝑥 < 13.8
(2) 𝐴𝑛𝑠𝑤𝑒𝑟 (4)(−5, 5)

(3) Answer (4)

Page 45 of 119
KVSRO/EKM/XI/MATHS

20) 1) 90 ≤ 𝐼𝑄 ≤ 150 ⇒ 90 ≤ 𝑀𝐴 × 100 ≤ 150


𝐶𝐴

𝑀𝐴
⇒ 90 ≤ × 100 ≤ 150
15
15 15
⇒ 90 × ≤ 𝑀𝐴 ≤ 150 ×
100 100
⇒ 13.5≤ 𝑀𝐴 ≤ 22.5
𝑀𝐴 𝐼𝑄×𝐶𝐴
2) 𝐼𝑄 = × 100 ⇒ 𝑀𝐴 =
𝐶𝐴 100
𝐼𝑄 × 𝐶𝐴
9 ≤ 𝑀𝐴 ≤ 15 ⇒ 9 ≤ ≤ 15
100
𝐼𝑄 × 12
⇒9≤ ≤ 15
100
100 100
⇒9× ≤ 𝐼𝑄 ≤ 15 ×
12 12
⇒ 75 ≤ 𝐼𝑄 ≤ 125

Page 46 of 119
KVSRO/EKM/XI/MATHS

Chapter 6
PERMUTATIONS AND COMBINATIONS
IMPORTANT FORMULA

Factorial Notation:
Let n be a positive integer. Then, factorial n, denoted n! is defined as:
n! = n(n – 1)(n – 2) … 3.2.1.
We define 0! = 1.
Examples:
4! = (4 x 3 x 2 x 1) = 24.

5! = (5 x 4 x 3 x 2 x 1) = 120.
Permutations:The different arrangements of a given number of things by taking some or all at a
time, are called permutations.
Examples:
All permutations (or arrangements) made with the letters a, b, c by taking two at a time are
(ab, ba, ac, ca, bc, cb).
All permutations made with the letters a, b, c taking all at a time are:
( abc, acb, bac, bca, cab, cba)
Number of Permutations:
Number of all permutations of n things, taken r at a time, is given by:
n!
nPr = n(n - 1)(n - 2) ... (n - r + 1) =
(n - r)!
Examples:
6P2 = (6 x 5) = 30.
7P3 = (7 x 6 x 5) = 210.
Number of all permutations of n things, taken all at a time = n!.
An Important Result:
If there are n subjects of which p1 are alike of one kind; p2 are alike of another kind; p3 are alike of
third kind and so on and pr are alike of rth kind,
such that (p1 + p2 + ... pr) = n.
n!
Then, number of permutations of these n objects is =
(p1!).(p2)! ... (pr!)

Page 47 of 119
KVSRO/EKM/XI/MATHS

Combinations:Each of the different groups or selections which can be formed by taking some or
all of a number of objects is called a combination.
Examples:

Suppose we want to select two out of three boys A, B, C. Then, possible selections are AB, BC and
CA.

Note: AB and BA represent the same selection.


i. All the combinations formed by a, b, c taking ab, bc, ca.
ii. The only combination that can be formed of three letters a, b, c taken all at a time
is abc.

iii. Various groups of 2 out of four persons A, B, C, D are:


AB, AC, AD, BC, BD, CD.
iv. Note that ab ba are two different permutations but they represent the same
combination.
Number of Combinations:
The number of all combinations of n things, taken r at a time is:
n! n(n - 1)(n - 2) ... to r factors
nCr = = .
(r!)(n - r)! r!
Note:
v. nCn = 1 and nC0 = 1.
vi. nCr = nC(n - r)
Examples:
(11 x 10 x 9 x 8)
i. 11C4 = = 330.
(4 x 3 x 2 x 1)
16 x 15 x 14 16 x 15 x 14
ii. 16C13 = 16C(16 - 13) = 16C3 = = = 560.
3! 3x2x1

Page 48 of 119
KVSRO/EKM/XI/MATHS

SUMMARY

MIND MAP

Page 49 of 119
KVSRO/EKM/XI/MATHS

Page 50 of 119
KVSRO/EKM/XI/MATHS

1 MARK QUESTIONS (1X 5)


1. How many 4 letter code can be formed using the first 10 letter of the English
alphabet, if no letter can be repeated?

2. Find The number of ways in which 8 distinct toys can be distributed among 5
children.
3. Find the number of ways in which 8 students can be sated in a line?
4. In how many ways can a group of 5 men and 2 women be made out of a total
of 7 men and 3 women?
5. A box contains 4 red, 3 white and 2 blue balls. Three balls are drawn at
random. Find out the number of ways of selecting the balls of different
colours?
6. A coin is tossed 6 times, and the outcomes are noted. How many possible
outcomescan be there?

2 MARK QUESTIONS (2 X 10)


7. Evaluate

8. A student is to answer 10 out of 13 questions in an examination such that he


must choose at least 4 from the first five questions. Find the number of
choices available to him.
9. In How many ways can final eleven be selected from 15 cricket players if
(i) One of them must be included
(ii) One of them, who are in bad form, must always be excluded.
10. How many words, with or without meaning each of 2 vowels and 3
consonants can be formed from the letter of the word DAUGHTER.
11. Find the number of positive integers greater than 6000 and less than 7000
which are divisible by 5, provided that no digit is to be repeated.
12. It is needed to seat 5 boys and 4 girls in a row so that the girl gets the even
places. How many are such arrangements possible?

Page 51 of 119
KVSRO/EKM/XI/MATHS

13. Find the number of 5-card combinations out of a deck of 52 cards if each
selection of 5 cards has exactly one king.
14. Find the 3-digit numbers that can be formed from the given digits: 1, 2, 3, 4
and 5 assuming thata) digits can be repeated.
b) digits are not allowed to be repeated.

15. 25 buses are running between two places P and Q. In how many ways can a

person go from P to Q and return by a different bus?

LONG ANSWER (3X 4)


16. If 2n C3: n C 3 = 11:1 find n.
17. Out of 6 boys and 4 girls a committee of 5 is to be formed. In how many ways
ear this be done it. (i) at least 2 girls are included
(ii) at most 2 girls are included.
18. A deck of 52 cards has 4 suits: diamonds, hearts, clubs, and spades.
a) What is the number of ways of choosing 4 cards from a pack of 52
playing cards if two are red cards & two are black cards.
b) What is the number of ways of choosing 4 cards from a pack of 52
playing cards if cards are of the same colour?

CASE STUDY
Rinu a 5th class child was playing a word game with her father. Her father asked her to find
all the possible words that can be made using the word CAT. She listed the words as
CAT, CTA, ATC, TCA, ACT, TAC, Now father told her to arrange them in alphabetical
order. She arranged it like this:ACT, ATC, CAT, CTA, TAC, TCA
The father asked her the position or Rank of the word CAT in this arrangement. She could
easily identify that CAT is third in the above list. Father explained her so, the rank of the
word CAT is 3 when you arrange all possible combinations of the letters. She asked the
father to find the rank of SUCCESS If all possible permutations of the word SUCCESS are
arranged in dictionary order.

Page 52 of 119
KVSRO/EKM/XI/MATHS

But, as you might have realized by now – the problem would become extremely difficult if
the word is bigger. Can you help Rinu to find out the following

19. The Number of words starting with C


a)60 b)80 c)120 d)180
20. Number of words starting with E
a)60 b)80 c)120 d)30
21. Number of words starting with SC
a)60 b)30 c)120 d)180

22. Find the rank of the word ‘SUCCESS’ in dictionary format.


a) 360 b)330 c)331 d)361

ANSWERS
1. First letter can be used in 10 ways
Second letter can be used in 9 ways
Third letter can be used in 8 ways
Forth letter can be used in 7 ways
By Fundamental principle of counting total no. of ways
2. Total number of toys = 8
Total number of children = 5
Now, each toy can be distributed in 5 ways.
So, total number of ways = 5 × 5 × 5 × 5 × 5 × 5 × 5 × 5= 585.
3. The number of ways in which 8 students can be sated in a line = 8P8
= 8!= 40320
4. We need to select 5 men from 7 men and 2 women from 3 women

Number of ways to do this


= 7C5 x 3C2
= 7C2 x 3C1 [Applied the formula nCr = nC(n - r) ]= (7×62×1)×3
= 21 x 3 = 631

Page 53 of 119
KVSRO/EKM/XI/MATHS

5. 1 red ball can be selected in 4C1 ways


1 white ball can be selected in 3C1 ways
1 blue ball can be selected in 2C1 ways
Total number of ways
= 4C1 x 3C1 x 2C1
=4 x 3 x 2
= 24
6. When we toss a coin once, the number of outcomes we get is 2 (Either Head or tail)
So, in each throw, the no. of ways to get a different face will be 2.
Therefore, by the multiplication principle, the required no. of possible outcomes is
2 x 2 x 2 x 2 × 2 × 2 = 64

7.

8. There are two cases


When 4 is selected from the first 5 and rest 6 from remaining 8
Total arrangement = 5C4 × 8C6
= 5C1 × 8C2
= 5 × (8×7)/(2×1)
=5×4×7
= 140
When all 5 is selected from the first 5 and rest 5 from remaining 8
Total arrangement = 5C5 × 8C5
= 1 × 8C3
= (8×7×6)/(3×2×1)
= 8×7
= 56
Now, total number of choices available = 140 + 56 = 196

9. (i) 14 C10= 1001(ii) 14 C11 = 364

Page 54 of 119
KVSRO/EKM/XI/MATHS

10. Total number of letters are 8

Total vowels =3 and consonants =5

Now we have to make a word using five letters from given word.

Here we have to choose 2 out of three vowels and 3 out of 5 consonants which can
further be arranged in 5! ways.3C2×5C3×5!

11. We have to form 4-digit numbers which are greater than 6000 and less than 7000.
We know that a number is divisible by 5, if at the unit place of the number there is 0
or 5.So, unit digit can be filled in 2 ways.
The thousandth place can be filled by ‘6’ only.
The hundredth place and tenth place can be filled together in 8 x 7 = 56 ways. So,
total number of ways = 56 x 2 = 112

12. 5 boys and 4 girls are to be seated in a row so that the girl gets the even places.
The 5 boys can be seated in 5! Ways.For each of the arrangements, 4 girls can be
seated only at the places which are cross marked to make girls occupy the even
places)B x B x B x B x BSo, the girls can be seated in 4! Ways.
Hence, the possible number of arrangements = 4! × 5! = 24 × 120 = 2880

13. Take a deck of 52 cards,To get exactly one king, 5-card combinations have to be
made. It should be made in such a way that in each selection of 5 cards, or in a deck
of 52 cards, there will be 4 kings.
To select 1 king out of 4 kings = 4C1
To select 4 cards out of the remaining 48 cards = 48C4
To get the needed number of 5 card combination = 4C1 x 48C4
= 4x2x 47x 46×45= 778320 ways
14. a) By the multiplication principle, the number of ways in which three-digit numbers can
be formed from the given digits is 5 × 5 × 5 = 125
b) By the multiplication principle, the number of ways in which three-digit numbers can
be formed without repeating the given digits is 5 × 4 × 3 = 60

Page 55 of 119
KVSRO/EKM/XI/MATHS

15. He can go in any bus out of the 25 buses.


Hence He can go in 25 ways.
Since he can not come back in the same bus that he used for travelling,
He can return in 24 ways.Total number of ways = 25 x 24 = 600
2𝑛!
𝑛!
16. Ans : =
3!(2𝑛−3)! 3!(𝑛−3)!
2(2𝑛−1)(2𝑛−2)(2𝑛−3)! (𝑛−1)(𝑛−2)(𝑛−3)!
=
3!(2𝑛−3)! 3!(𝑛−3)!
2(2𝑛−1)(2𝑛−2) 𝑛(𝑛−1)(𝑛−2)
=
3! 3!
2(2𝑛−1)(2𝑛−2) (𝑛−1)(𝑛−2)
= (on Solving )
3! 3!

18 -3n = 0 , n = 6
17. (I) including at least 2 girls, there are three cases
(a) 2 boys 3 girls, 4c2 x 6c3 = 120
(b) 3 girls and 2 boys, 4c3 x 6c2 = 60
(c) 4 girls and 1 boy, 4c4 x 6c1 = 6
 Total number of ways = 120+60+6= 186
(II) Including at most 2 girls.
(a) 5 boys no girl = 6c5 = 6c1 = 6
(b) 1 girl 4 boys = 4c1 x 6c4 = 60
(c) 2 girls 3 boys = 4c2 x 6c3 = 120
 Total number of ways = 6+60+120= 186

18. a)If 2 cards are red and 2 are black then. Out of 26 red card 2 cards can be selected

in ways similarly 2 black card can be selected in ways

required no. of selection

(b) If 4 cards are of the same colour each colour can be selected in ways
Then required no. of selection

Page 56 of 119
KVSRO/EKM/XI/MATHS

19. Alphabets present in the word SUCCESS are S,U,C,E


Dictionary order of the alphabet's is C,E,S,U
Number of words starting with C (C ______ )
6!
(no two C will be repeated but here we have three S) is = 120
3!

20.Number of words starting with E (E_______ )


6!
(here we have two C and three S) is = 60
2!3!

21. Now we want the word stariting with S


So, Number of words starting with SC (SC _____ )
5!
(here we have single C and two S) is =60
2!
5!
22. Number of words starting with SE (SE _ _ _ _ _) (here we have two C and two S) is =30
2!2!
5!
Number of words starting with SS (SS _____ ) (here we have two C and single S) is =60
2!

Now next word will be SUCCESS Now rank of the word SUCCESS is
⇒120+60+60+30+60+1=331.

Page 57 of 119
KVSRO/EKM/XI/MATHS

Chapter 7
BINOMIAL THEOREM
Multiple Choice Questions
Choose the correct answer in each of the following.
1) The value at the 6th row and 3rd place in the Pascal’s triangle is
(a) 5 (b)10 (c)15 (d)20
2) The sum of exponents of x and y in the expansion of (𝒙 + 𝒚)𝒏 is
(a) 𝒏𝟐 (b) n+1 (c) 2n (d) n
3) The number of terms in the expansion of (𝒙 + 𝒚)𝒏 is
(a) n+1 (b) 𝒏𝟐 (c) 2n (d) n
4) The total number of terms in the expansion of (𝒙 + 𝒂)𝟏𝟎𝟎 + (𝒙 − 𝒂)𝟏𝟎𝟎
(a) 50 (b) 200 (c) 51 (d) none of these
5) Find the number of terms in the expansion of [(𝟐𝒙 + 𝟑𝒚)𝟐]𝟓
(a) 11 (b) 10 (c) 5 (d) 6
Short Answer Questions

1) Using binomial theorem, expand ( 𝟑√𝒙 − 𝟑√𝒚)


2) Prove that (𝟐 + √𝟓) + (𝟐 − √𝟓)𝟒 = 𝟏𝟑𝟔𝟒
3) Using binomial theorem evaluate (i) 𝟏𝟎𝟏𝟒 and (ii) 𝟗𝟖𝟓
4) Find the coefficient of x in the expansion of (1-3x+7𝒙𝟐) (𝟏 − 𝒙)𝟏𝟔
5) Prove that𝟐𝟑𝒏 − 𝟕𝒏 − 𝟏 is divisible by 49 by using binomial theorem.
6) Prove that ∑𝒏𝒓=𝟎 𝟓𝒓 𝒏𝑪𝒓 = 𝟔𝒏
7) Find the coefficient of 𝒙𝟓 in (𝒙 + 𝟑)𝟖
8) Using binomial theorem, prove that 𝟔𝒏-5n always leaves the reminder 1 when
divided by 25.

Case Study Question


1) Two schools A & B join the campaign “Keep your place clean” as part of spreading
awareness in the public to keep their place and city clean. The number of students of
𝟏
school A&B who joined the campaign are 3rd term in the expression (𝟒𝒙 + )𝟒 and 4th
𝟐𝒙
𝟏 𝟔
term in the expression (𝒙 + ) . Find
𝒙
Find (i) number of students of school A who joined the campaign
(ii) number of students of school B who joined the campaign

Page 58 of 119
KVSRO/EKM/XI/MATHS

Long Answer Questions

1) The first three terms in the expansion of (𝟏 + 𝒂𝒙) where n is a positive integer
are 1 + 12x + 64𝒙𝟐 . Find n and a.

2) If P be the sum of odd terms and Q be the sum of even terms in the expansion of
(𝒙 + 𝒂)𝒏, prove that (i ) 𝑷𝟐 − 𝑸𝟐 = (𝒙𝟐 − 𝒂𝟐)𝒏

(ii ) 4PQ = (𝒙 + 𝒂)𝒏 − (𝒙 − 𝒂)𝒏


…………………………………………………………………………………………………………
ANSWERS
Multiple Choice Questions
1) B 2) D 3) A 4) C 5) A

Short Answer Questions


𝟒 𝟏
1) 𝒙𝟓/𝟑 − 𝟓𝒙𝟑𝒚𝟑 + 𝟏𝟎𝒙𝒚𝟐/𝟑 − 𝟏𝟎𝒙𝟐/𝟑𝒚 + 𝟓𝒙𝟏/𝟑𝒚𝟒/𝟑 − 𝒚𝟓/𝟑
2) By applying binomial theorem expand and simplify
3)i) 104060401 ii) 9039207968
4) -19
5)𝟑𝒏 − 𝟕𝒏 − 𝟏= (𝟐𝟑)𝒏-7n-1
=𝟖𝒏-7n-1
=(𝟏 + 𝟕)𝒏 -7n-1
= 1+ 𝒏𝑪 𝟏𝟕 + 𝒏𝑪 𝟐𝟕𝟐 +……………+𝒏𝑪 𝒏𝟕𝒏 -7n-1

= 𝒏𝑪 𝟐𝟕𝟐 +𝒏𝑪 𝟑𝟕𝟑……………+𝒏𝑪 𝒏𝟕𝒏

= 𝟕𝟐 (𝒏𝑪 𝟐 + 𝒏𝑪 𝟑𝟕𝟏……………+𝒏𝑪 𝒏𝟕𝒏−𝟐)

𝑺𝒐 𝟐𝟑𝒏 − 𝟕𝒏 − 𝟏is divisible by 49.6)𝒏


= (𝟏 + 𝟓)𝒏
= 𝒏𝑪𝟎𝟓𝟎𝟏𝒏 + 𝒏𝑪 𝟏 𝟓𝟏𝟏𝒏−𝟏 + 𝒏𝑪𝟐𝟓𝟐𝟏𝒏−𝟐+………………………+𝒏𝑪𝒏𝟓𝒏𝟏𝟎

= 𝒏𝑪𝟎 𝟓𝟎 + 𝒏𝑪𝟏 𝟓𝟏 + 𝒏𝟐𝟓𝟐+………………………+𝒏𝑪𝒏𝟓𝒏 = ∑𝒏𝒓=𝟎 𝟓𝒓 𝒏𝑪𝒓

Page 59 of 119
KVSRO/EKM/XI/MATHS

7)𝑪𝟓 = 56
8) 𝟔𝒏 = (𝟏 + 𝟓)
= 𝒏𝑪𝟎𝟓𝟎𝟏𝒏 + 𝒏𝑪 𝟏 𝟓𝟏𝟏𝒏−𝟏 + 𝒏𝑪𝟐𝟓𝟐𝟏𝒏−𝟐+………………………+𝒏𝑪𝒓𝟓𝒓𝟏𝟎

= 1+5n +𝒏𝑪𝟐𝟓𝟐𝟏𝒏−𝟐+ ..........................+𝒏𝑪𝒏𝟓𝒏𝟏𝟎


𝟔𝒏 − 𝟓𝒏 = 1+𝒏𝑪𝟐𝟓𝟐𝟏𝒏−𝟐+………………………+𝒏𝑪𝒏𝟓𝒏𝟏𝟎
= 1+25(𝒏𝑪𝟐+………………………+𝒏𝑪𝒏𝟓𝒏−𝟐)
=1+ 25xpositive integer
Hence 𝟔𝒏 − 𝟓𝒏 always leaves the remainder 1 when divided by 25

Case study Question


1) (i) 24
(ii) 20
Long Answer Questions
1) 𝒏𝑪𝟏a =12, 𝒏𝑪𝟐𝒂𝟐 = 64
On simplifying n=9, a=4/3
2)(i)(𝒙 + 𝒂)𝒏 = 𝒏𝑪𝟎 𝒂𝟎𝒙𝒏 + 𝒏𝑪𝟏 𝒂𝟏𝒙𝒏−𝟏 + 𝒏𝑪𝟐𝒂𝟐𝒙𝒏−𝟐+ ................. +𝒏𝑪𝒏𝒂𝒏𝒙𝟎

=P+ Q
(𝒙 − 𝒂)𝒏 = 𝒏𝑪𝟎 𝒂𝟎𝒙𝒏 - 𝒏𝑪𝟏 𝒂𝟏𝒙𝒏−𝟏 + 𝒏𝑪𝟐𝒂𝟐𝒙𝒏−𝟐+………………-𝒏𝑪𝒏𝒂𝒏𝒙𝟎

= P-Q
(𝒙 + 𝒂)(𝒙 − 𝒂)𝒏 = (P+Q)(P-Q)
(𝒙𝟐 − 𝒂𝟐)𝒏 = 𝑷𝟐 − 𝑸𝟐
(ii)4PQ = (𝑷 + 𝑸)𝟐 − (𝑷 − 𝑸)𝟐
= (𝒙 + 𝒂)𝟐𝒏 - (𝒙 − 𝒂)𝟐𝒏
……………………………………………………………………………………………………………

Page 60 of 119
KVSRO/EKM/XI/MATHS

Chapter 8
SEQUENCES AND SERIES

CONCEPT AND MIND MAPPING

MULTIPLE CHOICE QUESTIONS(1 MARK)

1. Which term of the sequence 2, 2 2 ,4 , ………is 128?


a)12 b) 13 c) 10 d) 9

Page 61 of 119
KVSRO/EKM/XI/MATHS

2. For what value of x,the numbers -2/7,x,-7/2 are in GP


a)1 b) -1 c) ±1 d) 2
3. The12th term of a G .P Whose 8th term is 192 and the common ratio 2 is

a) 3082 b) 3027 c) 3702 d) 3072


4. Find The 35 th term of the sequence 4,0,-4,-8,…. is

a)-132 b) 132 c) 134 d) None of these


5. If k+2,4k-6 and 3k-2 are the three consecutive terms of an AP.The value of k is

a)1 b) -1 c) ±1 d) 3

SHORT ANSWER QUESTIONS (2 MARKS)


6. If the sum of a certain number of terms of the A.P. 25,22,19,…is 116 , find the last term

7.In an AP the first term is 2 and the sum of the first five terms is one fourth of the next five
terms. Find the common difference.
8. If the AM and GM of two positive numbers a and b are 10 and 8 respectively, find the

numbers.
9. If p times the pth term of an AP is q times the qth term.Find the (p+q)th term.
10. For what value of n,the nth terms of the sequences 3,10,17,…and 63,65,67,…are equal
11. The third term of an AP is p and 4 th term is q.Find the 10th term.

12. If 7 times 7th term of an AP is 11 times its 11 th term.Find its 18 th term


13. If pth and q th terms of an AP are 1/pr and 1/qr respectively,find the rth term
14.Find the number of terms in the series 5+8+11+…if the last term is 95
15.The first term of a GP is 1.The sum of 3rd and fifth terms is 90.Find its common ratio
16.The sum of first 3 terms of a GP is to sum of first 6 terms is125:152. Find its common ratio
17.The AM of two numbers exeeds their GM by 2 and the ratio of the numbers is 4.Find the
numbers.
18. If the 4th term of an AP is 13 ,then find sum of first seven terms
19.The AMand GM of roots of a quadratic equation are 8 and 5respectively.Find the quadratic
equation
20. Find the sum of 20 terms of the GP 0.15, 0.015, 0.0015…

Page 62 of 119
KVSRO/EKM/XI/MATHS

CASE STUDY QUESTIONS

21.A polygon is regular when all angles are equal and all sides are equal (otherwise it is
"irregular"). Below given figure is an equilateral triangle with sides 18cm.The midpoints of its
sides are joined to form another triangle whose midpoints ,in turn ,are joined to form another
triangle .The process is continued indefinitely. Answer the questions given below:

i).The sum of perimeters of all the triangles will be:


a)100 cm b) 110cm c) 118cm d)108cm
ii).The sum of areas of all the triangles is
a) 105 √3 cm b) 108√3 c) 2 108cm d) 2
iii) The sequence of lengths of sides of a triangle form a/an
a)AP b)GP c) Harmonic progression d) None of these
iv) The sum to infinity of a geometric series where first term is a and common ratio r is given by
a) a/1-rb) (1-r)/ac) 1-r d) None of these

22. The side of a given square is 10 cm.The midpoints of its sides are joined to form a new

square.Again,the midpoints of the sides of the new square are joined to form another square.The
process is continued indefinetly.based on the above information answer the questions below:

Page 63 of 119
KVSRO/EKM/XI/MATHS

i) The sum of areas of all the squares so formed will be


a)300 sqcm b) 350 sqcm c) 200 sqcm d) 400 sqcm
ii) The sum of Perimeters of the squares so formed will be
a) 40+ 80√2 cm b) 80 +40 √2 cm c) 80√2 cm d) 40√2 cm

23. We need to grow more trees to make our surroundings better and to compensate for the
deforestation and reducing effects of air pollution. To better our individual and social health let us
grow more trees. In a village there are 30 trees at equal distances of 5 meters in a line around a
well. The distance of the well from the nearest tree being 10 meters. A Gardner waters all the
trees Separately starting from the well and returns to the well after watering each tree to get
water for the next. Now answer the questions :

i) The total distance covered ( in meters) by the gardner is:


a)4975m b)4795m c)4955m d)4275m
ii) The terms having constant difference is called as:
a) Difference b)common ratio c)Common difference d)none
iii) The number of A.P’s containing 10 terms in which the first term is in the set { 1,2,3} and the
common difference is in the set {2,3,4} will be :
a)10 b)9 c)6 d)8

Page 64 of 119
KVSRO/EKM/XI/MATHS

24. 24.

Chessboard is the type of game board used for the game of chess, on which the chess pawns and
pieces are placed. A chessboard is usually square in shape, with an alternating pattern of squares
in two colours, with its side being divided into eight parts, resulting in a total of 64 squares. The
inventor of the chess board suggested a reward of one gram of wheat for the first square,2 grams
for the second,4 grams for the third and so on .doubling the number of grains for for subsequent
squares .Based on the above information answer the questions given below:
i) . How many grains would have to be given to the inventor ?
a)1024 b) 264 -1 c) 263 -1 d) 264
ii) .The number of grains in each square forms a /an
a)Arithmetic sequence b)Harmonic sequence c)Geometric sequence d)None
iii). The sum to n terms of Arithmetic series is given by
a) Sn= a/1-r b) Sn= 1-r/a c) n/2[2a+(n-1)d] d) none of these
iv).The sum to n terms of Geometric series is given by
a) Sn= a/1-r b) Sn= a(1-rn)/1-r,r<1 c) n/2[2a+(n-1)d] d) none of these

Long Answer Questions


25. The sum of the first four terms of an A.P is 56. The sum of the last four terms is 112. If its

first term is 11, find the no of terms.


26. The sum of the first three terms of a GP is 7and sum of their squares is 21.Determine the
first 5 terms of the GP
27. If a,b,c are in AP and x,y,z are in GP., then show that xbc .yca .zab  1
28. The sum of n terms of two APs are in the ratio (7n + 1):(4n + 27). Find the ratio of the
13th terms.
29. The sum of four numbers in GP is 60 and the AM of first and last is 18.Find the numbers

Page 65 of 119
KVSRO/EKM/XI/MATHS

ANSWERS
MCQ
1.b 2.c 3. d 4.a 5.d

SHORT ANSWER QUESTIONS (2 MARKS)


6.LAST TERM=4 7.d= - 6 8.16 and 4 9. 0 10.13 11. 7q-6p 12. 0
13. 1/pq 14.31 15.±3 16.3/5 17. 16 and 4 18. 91
19.x2- 16x + 25 = 0 20. 1/6[1-(.1)20]
CASE STUDY QUESTIONS

21 i)d ii)b iii)b iv)a 22 i)c ii)b


23 i)b ii)c iii)b 24) i)b ii)c iii)c iv)b

LONG ANSWER QUESTIONS


25. S4 = 56, Sn – Sn-4 = 112, Solving n=11

26.1,2,4,8,16 OR 4,2,1,1/2,1/4( The terms can be taken as a,ar,ar2)


27.
ca
xbc .y c a .zab  xbc . xz .zab
2b(ac ) (ac )2b

x 2
.z 2

 x 0 .y 0  1

28.

29. The value of r=2 or ½,a=4


The numbers are 4,8,16,32 or 32,16,8,4
The equations can be taken as a+ar+ar2+ ar3=60 and a+ar3=36

Page 66 of 119
KVSRO/EKM/XI/MATHS

Chapter 9
STRAIGHT LINES
MCQ
1. The slope intercept form of the equation of line 2x + 3y -12 = 0 is
a. y = 𝑥 + 12
6
b. y = -2𝑥 + 6
3
c. y = 2𝑥 + 12
3
d. y = -2𝑥 + 4 Ans: y = -2𝑥 + 4
3 3

2. The equation of the line passing through a point (2,3) and parallel to x-axis is
a. y – x = 5
b. x = 2
c. y = 3
d. x + y = 5 Ans: y=3

3. The slope of a line with angle of inclination 120 0 is


a. 0
b. -√3
c. 1
d. 1 Ans: -√3
√3
4. The x- intercept of the line 3x + 7y – 6 = 0 is
a. 3
b. 2
c. -3
d. 1 Ans: 2
5. The equation of line with slope 2 and cutting the y-axis 3 units belowthe origin is
a. Y – 2x = 3
b. Y + 2x = 3
c. Y – 2x = - 3
d. None of the above Ans : Y – 2x = 3

The followingquestionsare of 2 marks each


1) Find a point on the y axis which is equidistant from (3,4) and (7,6)
Ans: (0,15)
2) Find the value of ‘ a ‘ so that 3 is the slope of the line through the point ( a ,4) and (5,-2)
(ans. a = 11)
3) Show that the line joining (2,-5) and (-2,5) is perpendicular to the line joining (6,3) & (1,1)
4) Reduce the equation 4x – 7y + 12 = 0 into intercept form and find their intercepts on the axes
Ans: 𝑥 + 12𝑦
=1
−3
7

5) Find the distance between the parallel lines 7x – 3y + 6 = 0 and 7x -3y -5 = 0 ( Ans: 11 )
√58

Page 67 of 119
KVSRO/EKM/XI/MATHS

6) Find the equation of the line passing through the point (1,-3) and inclined at an angle of 2250(Ans: -x
+ y = -4)
7) Find the equation of a line which makes an angle of tan-1(2) with the x-axis and cuts off an intercept
of 7 units on negative direction of y-axis (Ans : y – 2x +7 = 0)
8) Determine the equation of the line through the point (6, -2) which is parallel to the line 3x - 4y
=1
9) Find the distance of the point (2,5) from the line 8x -6y + 5 = 0 (Ans: 9 )
10
10) Find the equation of a line which divides the join of (0, 3.5) and (0,7) in the ratio 3:2 and
perpendicular to it ( Ans: y = 4)

Case Study questions: the following questions are case study questions
Advertising has many advantages. In modern business world, advertising plays an important role to
establish contact between the buyer and seller , There are 2 main types of outdoor advertising:
traditional and digital.Traditional types of outdoor advertising include billboards and posters.

1)

A rectangular board is used for advertisement and one side of the rectangle lies along the
line 4x + 7y + 5 =0. Two of it’s vertices are (-3,1) and (1,1)
Based on the above information answer the following
a) Find the equation of the side opposite to 4x + 7y +5 = 0 , if (1,1) lie on the side. (2 marks)
b) What is the slope of the given line (1 mark )
c) Find the slope of the adjacent side of the of the board (1 mark )

Solution : a.) Let the side opposite to 4x + 7y +5 = 0


is 4x + 7y + K = 0 ------------------- (i)
since (1,1) lie on the side so put x= 1,y=1 in (i)
⸫ K = -11
So the required equation of the side is 4x + 7y -11 = 0
4
b) the slope of the given line is −
7
c) the slope of the adjacent side is 7
4
2. We all like to spend time in parks Many of our childhood memories happened in parks. To some extent,
we’re probably aware that the parks in the neighbourhood where you grew up had an impact on who you
are today. We can see many mathematical concepts are there in park

Page 68 of 119
KVSRO/EKM/XI/MATHS

For example there is an equilateral triangular shaped board is kept in a park if the equation of the base of
the triangle is x + y = 2 and the opposite vertex has coordinates ( 2, 1 )

Based on the above solve the following

a) What is the length of the altitude drawn from the vertex (2,1) to the base
b) What is the area of the board A (2,1)
Solution : a) the equation of the base BC is x + 2y = 2
2 + 2−2
The length of the altitude AD is = | | = √2
√12+12

b.) sin 600 = 𝐴𝐷 = √2


B D C
𝐴𝐵 𝐴𝐵

√3 √2
=
2 𝐴𝐵

AB = 2√2= side
√3
1 1 2√2 2
Area of the triangle = . 𝐴𝐷. 𝐵𝐶= × ×√2= sq units
2 2 √3 √3

Long answer type questions


1. find the coordinates of the foot of perpendicular from the point (2, 1) to the line x – 3y = 2
1
Ans: coordinates of the foot of perpendicular is ( 23 , )
10 10

2. find the equation of the right bisector of the line segment joinining the points (2,3) and (4,2) Ans:4x-2y=7
3. find the equation of the lineparallelto x – 5y + 8 = 0 and which is such that the sum of its intercepts on
the axesis 7

Ans: x - 5y - 35 =0
4

4. the Fahrenheit temperature F and absolute temperature K satisfy a linear equation. Given that
K= 331 when F = 39 and that K = 202 when F = 21 express K in terms of F. based on this find
a) F when K = 300
b) Find K when F = 0
Ans: K = 43 ( F -39) +331
6

Page 69 of 119
KVSRO/EKM/XI/MATHS

a) F = 34.67
1331
b) K =
2
5 . find the area of the quadrilateral ABCD with the vertices A(1,0) , B(-2,3) , C(1,3) and D ( 5 , 2)

Ans: 21 sq. unit


2

******************************************************************************

THINGS TO BE REMEMBER:

FORMULAE: 1. Slope of a non vertical line passing through the points (x1 , y1) ,(x2 , y2) is
𝒚𝟐−𝒚𝟏
m=
𝒙𝟐−𝒙𝟏
2. Two non vertical lines are perpendicular to each other if and only if the product of
their slopes m1 . m2 = -1

3. Equation of a line in point slope form isy-y1 = m(x-x1) , where (x1,y1) is a fixed
Point and ‘m’ is the slope of the line
𝒚𝟐−𝒚𝟏
4. Equation of a line in two point form is : y - y1 = (x-x 1)
𝒙𝟐−𝒙𝟏
5. Equation of a line in slope intercept form is : y = mx + c where y-intercept is c
𝒚
6.Equation of a line in intercept form is :𝒙 + =𝟏
𝒂 𝒃

7. distance of a point from a line ; the distance (d) of a line Ax + By+ C =0 from a
|𝑨𝒙𝟏+𝑩𝒚𝟏+𝑪|
point P(x1,y1) is d =
√𝑨𝟐+𝑩𝟐

8. distance between two parallel lines : if Ax+By + C1 and Ax + By + C2are two

Parallel lines ,then the distance between them is d = |𝑪𝟏− 𝑪𝟐|


√𝑨𝟐+𝑩𝟐

9. Area of a triangle whose vertices are (x1,y1) , (x2,y2) and (x3 , y3) is
𝟏
Equal to │x1(y2 – y3) + x2(y3 – y1) + x3(y1-y1)│
𝟐

*****************************************************************************

Page 70 of 119
KVSRO/EKM/XI/MATHS

Chapter 10
CONIC SECTIONS

SECTION OF A CONE

A curve, generated by intersecting a right circular cone with a plane is termed as ‘conic’. It has
distinguished properties in Euclidean Geometry.. The vertex of the cone divides it into two nappes
referred to as the upper nappe and the lower nappe.

Page 71 of 119
KVSRO/EKM/XI/MATHS

When a cone is intersected by a plane, the section so obtained is known as conic section.

Depending upon the position of the plane which intersects the cone and the angle of intersection,
different types of conic sections are obtained. Namely;

 Circle
 Ellipse
 Parabola
 Hyperbola

CIRCLE : Circle is the set of all points in a plane that are equidistant from a fixed
point in the plane. The fixed point is called the centre of the circle and constant distance is the radius
of the circle.

Standard equation of circle

Equation of a circle with centre at (h,k) and radius r is (𝒙 − 𝒉)𝟐 + (𝒚 − 𝒌 )𝟐 = 𝒓𝟐

Equation of circle with centre at origin and radius r is x2 + y2 = r2

Page 72 of 119
KVSRO/EKM/XI/MATHS

PARABOLA : Parabola is the set of all points in a plane that are equidistant from a fixed

line and a fixed point in the plane. The fixed point is called focus and fixed line is known as
Directrix of the parabola.
Standard Equations of Parabola
1. y2 = 4ax

2. y2 = - 4ax
3. x2 = 4ay

4. x2 = - 4ay

Sl.no Standard form of Axis Vertex Focus Eqn of Directrix Length of


the parabola Latus rectum
1 Y2 = 4ax X - axis (0,0) (a,0) X+a = 0 4a
2 Y2 = - 4ax X - axis (0,0) (- a,0) x-a = 0 4a
3 X2 = 4ay Y - axis (0,0) (0,a) Y+a = 0 4a
4 X2 = - 4ay Y - axis (0,0) (0, -a) y-a =0 4a
ELLIPSE : - Ellipse is the set of all points in a plane , the sum of whose distances from two fixed
points in the plane is a constant. The two fixed points are called foci of the ellipse
Standard Equations of Ellipse

𝒙𝟐 𝒚𝟐
1.
𝒂𝟐
+ 𝒃𝟐 = 𝟏

𝒙𝟐 𝒚𝟐
2.
𝒃𝟐
+ 𝒂𝟐 = 𝟏

Page 73 of 119
KVSRO/EKM/XI/MATHS

Sl.no Equation of Relation Foci Vertices Eccentricity Length of Length of Length of


Ellipse between a,b,c major minor axis Latus
axis rectum
1 𝒙𝟐 𝒚𝟐 𝒂𝟐 = 𝒃 𝟐 + 𝒄 𝟐 (±𝒄, 𝟎) (±𝒂, 𝟎) 𝒄 2a 2b 𝟐𝒃𝟐
𝒂 𝟐+ 𝒃𝟐
=𝟏 𝒂 𝒂

2 𝒙𝟐 𝒚𝟐 𝒂𝟐 = 𝒃 𝟐 + 𝒄 𝟐 (𝟎, ±𝒄) (𝟎, ±𝒂) 𝒄 2a 2b 𝟐𝒃𝟐


𝒃 𝟐+ 𝒂𝟐
=𝟏 𝒂 𝒂

HYPERBOLA :Hyperbola isthe set of all points in a plane , the difference of whose
distances from two fixed points in the plane is a constant.
Standard Equations of Hyperbola

𝒙𝟐 𝒚𝟐
1.
𝒂𝟐
− =1
𝒃𝟐

𝟐 𝒙𝟐
2. 𝒚 − =1
𝒂𝟐 𝒃𝟐

Sl.no Equation of Relation Foci Vertices Eccentricity Length Length of Length of


Ellipse between a,b,c of major minor axis Latus
axis rectum
1 𝒙𝟐 𝒚𝟐 𝒄𝟐 (±𝒄, 𝟎) (±𝒂, 𝟎) 𝒄 2a 2b 𝟐𝒃𝟐
− = 𝒂𝟐 + 𝒃 𝟐 𝒂
𝒂 𝟐 𝒃𝟐 𝒂
=𝟏

2 𝒚𝟐 𝒙𝟐 𝒄𝟐 (𝟎, ±𝒄) (𝟎, ±𝒂) 𝒄 2a 2b 𝟐𝒃𝟐


− = 𝒂𝟐 + 𝒃 𝟐 𝒂
𝒂 𝟐 𝒃𝟐 𝒂
=𝟏

Page 74 of 119
KVSRO/EKM/XI/MATHS

MIND MAP

MULTIPLE CHOICE QUESTIONS

1. The equation of a circle with centre at (2,1) and area is 9𝝅 sq.unit is


(a) X2+ y2 -2x -4y +5 = 0 (b) x2 + y2 + 4x+4y +4 = 0
( c) x2 + y2 - 4x-2y - 4 = 0 (d) x2 + y2 -4x-2y -5 = 0

2. Equation of a parabola with vertex at origin, axis X axis and passing through the
point (-2,4) is
(a) Y2 = 4x (b) y2 = -4x (c) y2 = 8x (d) y2 = -8x
3. The eccentricity of the ellipse whose major axis is three times the minor axis is:

Page 75 of 119
KVSRO/EKM/XI/MATHS

(a) √𝟐 (b) √
𝟑 (c) (d) 𝟐
𝟑 𝟐 𝟐√𝟐 √𝟑
𝟑
2
4. The eccentricity of the hyperbola 9y - 4x 2= 36 is
(a) 𝟏𝟑 (b) 𝟏𝟑 (c) (d)√𝟓
𝟒 𝟐 √𝟏𝟑 𝟐
𝟐
5. If a parabolic reflector is 20cm in diameter and 5cm deep then the focus is:
(a) (2,0) (b) (3,0) (c ) (4,0) (d) (5,0)
6. The radius of the circle 2x2 + 2y2 – 8x -16y -10 = 0 is

(a) 3 (b) √𝟕 (c) 5 (d) √𝟐𝟎

7. The equation of the ellipse whose vertices are (± 13, 0) and

foci are (± 5, 0).


𝟐 𝟐 𝟐 𝟐 𝟐 𝟐 𝟐
(a) 𝒙 + 𝒚 =𝟐 𝟏 (b) 𝒙 +𝒚 = 𝟏 (c) 𝒙 + = 𝟏 (d) 𝒙 + 𝒚 =
𝒚
𝟏𝟔𝟗 𝟏𝟒𝟒 𝟏𝟔𝟗 𝟐𝟓 𝟐𝟓 𝟏𝟒𝟒 𝟏𝟒𝟒 𝟏𝟔𝟗

8. The length of latus rectum of the hyperbola y2 – 16 x2 = 16 is

(a) 3 (b) 𝟏 (c) 1 (d) 𝟐


𝟐 𝟑

9. The area of the triangle formed by the lines joining thevertex of

the parabola x2 = 12y to the ends of its latus rectum.

(a) 12 (b) 15 (c) 10 (d) 18


10. The centre of the circle x + y2 -2x =0 is
2

(a) (0,0) (b) (-1,0 ) (c) (0,1) (d) (1, 0 )

SHORT ANSWER TYPE QUESTIONS

1. Find the centre and radius of the circle x2 + y2 – 4x +6y =12


2. Find the equation of the circle having (1,-2) as its centre and passing through the
point of intersection of the lines 3x + y =14 and 2x + 5y = 18 .
3. Find the equation of the circle, the coordinates of whose diameters are (-1,2) and
(4, -3)
4. Find the coordinates of the focus, vertex, equation of directrix, length of latus
rectum of the parabola x2 = - 8y
5. Find the equation of the parabola with vertex at origin,axis X axis and passing
through the point (1,4)
6. Find the equation of the ellipse whose vertices are (0,±13) & the foci are(0,±5).
7. Find the equation of hyperbola whose length of latus rectum is 36 & foci are
(0,±12).
8. Find the equation of an ellipse whose foci are (±6,0) & eccentricity is 𝟑
𝟒

Page 76 of 119
KVSRO/EKM/XI/MATHS

9. Find the equation of the hyperbola whose conjugate axis is 5 and the distance
between the foci is 13 and axis is X axis.
10. Find the eccentricity of the hyperbola, the length of whose conjugate axis is 𝟑 of
𝟒
the length of transverse axis.
𝟐𝟖
11. Find the equation of the hyperbola whose vertices are (𝟎, ±𝟕) and foci at (𝟎, ± )
𝟑
12. Find the equation of the ellipse whose length of major axis is 26 and foci (𝟎, ±𝟓)
13. Find the eccentricity of the ellipse whose major axis is three times the minor
axis.
14. Find the equation of the ellipse with its foci on y axis, eccentricity 𝟑, centre at
𝟒
origin and passing through (6,4)
15. Find the equation of the circle with radius 5 whose centre lies on x-axis and
passes through the point (2,3).

LONG ANSWER QUESTIONS


1. Find the equation of the circle which passes through the points (3,7) and (5,5)
and has its centre on the line x-4y = 1.
2. The towers of a bridge, hung in the form of a parabola , have their tops 30m
above the roadway and 200m apart. If the cable is 5m above the roadway at the
centre of the bridge, find the length of the vertical supporting cable 3om from
the centre.
3. Find the coordinates of the foci,vertices, length of major and minor axis,
eccentricity and length of latus rectum of the ellipse 4x2 + 9y2 =1
4. An arch is in the form of a semi ellipse. It is 8m wide and 2m high at the
centre. Find the height of the arch at a point 1.5m from one end.
𝟐 𝒚𝟐
5. The foci of the hyperbola coincide with the foci of the ellipse 𝒙 + = 𝟏. Find
𝟐𝟓 𝟗
the equation of the hyperbola, if its eccentricity is 2
6. Find the coordinates of the foci, vertices, length of transverse and conjugate
axis, eccentricity and length of latus rectum of the hyperbola16x2- 9y2 =-144.
7. An equilateral triangle is inscribed in the parabola y2= 4ax so that oneangular
point of the triangle is the vertex of the parabola. Find thelength of the side of
the triangle.
8. Find the equation of the ellipse with centre at the origin, major axis ony-axis and
passing through the point (3,2) & (1,6).
9. A man running a race course notes that the sum of the distances fromtwo flag
posts from him is always 10m and distance between the flagpost is 8m. Find the
equation of the path traced by the man.
10. Find the equation of the hyperbola whose foci are (0, ± 10 ) & which

passes through the point (2, 3).

Page 77 of 119
KVSRO/EKM/XI/MATHS

CASE STUDY QUESTIONS


1. The cable of a uniformly loaded suspension bridge hangs in the form of a
parabola. The roadway which is horizontal and 100 m long is supported by
vertical wires attached to the cable, the longest wire being 30 m and the
shortest being 6 m.

(1) . Find the equation of the parabola


(2) Find the length of a supporting wire attached to the roadway 18 m from
the middle.
2. A farmer wishes to install two hand pumps in his field for watering.
The farmer moves in the field while watering in such a way that the

sum of the distances between the farmer and each hand pump is

26meters. Also, the distance between the hand pumps is 10 meters.

Page 78 of 119
KVSRO/EKM/XI/MATHS

(1) Curve along which the farmer moves is --------


(2) Find the equation of the curve traced by the farmer.
(3) Find the eccentricity of the curve along which the farmer moves

3. Indian track and field athlete Neeraj Chopra who competes in the
javelin throw, won a gold medal at Tokyo Olympics. He is the first
track and field athlete to win a gold medal for India at Olympics.

(1) Name the shape of the path followed by the javelin


(a) Half ellipse (b) Parabola (c ) Hyperbola (d) None of these

(2) If the equation of such a curve is given by x2 = -8y , then the


coordinate of focus is

Page 79 of 119
KVSRO/EKM/XI/MATHS

(a) (4,0) (b) (0,4) (c) (0,-2) (d) (-2,0)

(3) Equation of directrix is


(a) x +4 = 0 (b) x-4 =0 (c) x+2 = 0 (d) x-2 = 0

(4) Length of latus rectum is


(a) 4 (b) 8 (c) 2 (d) 16

ANSWERS

MULTIPLE CHOICE QUESTIONS

1.c 2.d 3.c 4.c 5.d


6. c 7.a 8.b 9.d 10.d

SHORT ANSWER TYPE QUESTIONS

1. centre (2,-3) radius = 5


2. x2 + y2 -2x +4y -20 =0
3. x2 + y2 -3x +y -10 =0
4. focus (0,-2) ,vertex (0,0) ,equation of directrix y-2 =0 ,length of
latus rectum = 8.
𝒙𝟐 𝒚𝟐 𝒚𝟐 𝒙𝟐
5. Y2 = 16x 6.𝟏𝟒𝟒 + 𝟏𝟔𝟗 = 𝟏 7.𝟑𝟔 − 𝟏𝟎𝟖 = 𝟏
𝒙𝟐 𝒚𝟐 𝒙𝟐 𝟒𝒚𝟐 𝟓
8.𝟔𝟒 + 𝟐𝟖 = 𝟏 9.𝟑𝟔 − =𝟏 10.𝟒
𝟐𝟓
𝒚𝟐 𝟗𝒙𝟐 𝒙𝟐 𝒚𝟐 𝟐√𝟐
11.𝟒𝟗 − 𝟑𝟒𝟑 = 𝟏 12.𝟏𝟒𝟒 + 𝟏𝟔𝟗 = 𝟏 13. 𝟑
𝒙𝟐 𝟕𝒚𝟐
14.𝟒𝟑 + 𝟔𝟖𝟖 = 𝟏15. X2 + y2 +4x -21 =0 , X2 + y2 -12x -1 =0 ,

Page 80 of 119
KVSRO/EKM/XI/MATHS

LONG ANSWER TYPE QUESTIONS

1. X2 + y2 +6x+2Y -90 =0 ,
𝟐𝟗
2. 𝟒
√𝟓 𝟏 √𝟓
3. Foci (± , 𝟎) vertices (± 𝟐 , 𝟎) e =
𝟑 𝟑
Length of major axis = 1
Length of minor axis = 2/9
Length of latus rectum = 4/9
4.
√𝟑𝟗
𝟒
𝒙𝟐 𝒚𝟐
5. − 𝟏𝟐 = 𝟏
𝟒
𝟓
6. Foci (𝟎, ±𝟓) vertices (𝟎, ±𝟒, 𝟎) e = 𝟒
Length of transverse axis = 8
Length of conjugate axis = 6
Length of latus rectum = 9/2
𝟖𝒂
7.
√𝟑
𝒙𝟐 𝒚𝟐
8. + 𝟒𝟎 = 𝟏
𝟏𝟎
𝒙𝟐 𝒚𝟐
9. + =𝟏
𝟐𝟓 𝟗
𝒚𝟐 𝒙𝟐
10. − =𝟏
𝟓 𝟓

CASE STUDY QUESTIONS


1. (1) 6x2 =625y (2) 3.11
𝒙𝟐 𝒚𝟐 𝟓
2. (1) ellipse (2) + 𝟏𝟒𝟒 = 𝟏 e =𝟏𝟑
𝟏𝟔𝟗
3. (1) parabola (2) (0,-2) (3) x-2 = 0 (4) 8

Page 81 of 119
KVSRO/EKM/XI/MATHS

Chapter 11
INTRODUCTION TO THREE-DIMENSIONAL GEOMETRY
(Syllabus: Coordinate axes and coordinate planes in three dimensions. Coordinate of a point.
Distance between two points)
Summary:
Coordinate axes: In three-dimensionalgeometry, the coordinate axes of a rectangular cartesian
coordinate system are three mutually perpendicular lines. The axes are called the 𝐱 −
𝐚𝐱𝐢𝐬, 𝐲 − 𝐚𝐱𝐢𝐬 𝐚𝐧𝐝 𝐳 − 𝐚𝐱𝐢𝐬.
Coordinate Planes: The three planes determined by the pair of axes are the coordinate planes,
called 𝐗𝐘, 𝐘𝐙 𝐚𝐧𝐝 𝐙𝐗 𝐩𝐥𝐚𝐧𝐞𝐬.
𝐄𝐪𝐮𝐚𝐭𝐢𝐨𝐧 𝐨𝐟 𝐗𝐘 𝐩𝐥𝐚𝐧𝐞 𝐢𝐬 𝐳 = 𝟎.
𝐄𝐪𝐮𝐚𝐭𝐢𝐨𝐧 𝐨𝐟 𝐘𝐙 𝐩𝐥𝐚𝐧𝐞 𝐢𝐬 𝐱 = 𝟎.
𝐄𝐪𝐮𝐚𝐭𝐢𝐨𝐧 𝐨𝐟 𝐙𝐗 𝐩𝐥𝐚𝐧𝐞 𝐢𝐬 𝐲 = 𝟎.
OCTANTS:The three coordinate planes divide the space into eight parts known as OCTANTS.
Coordinates of points in 3D : The coordinates of a point P in three dimensional geometry is
always written in the form of triplet like (x, y, z).Here
x, y and z are the distances from the YZ, ZX and XY planes.
𝐓𝐡𝐞 𝐜𝐨𝐨𝐫𝐝𝐢𝐧𝐚𝐭𝐞𝐬 𝐨𝐟 𝐭𝐡𝐞 𝐨𝐫𝐢𝐠𝐢𝐧 𝐢𝐬 (𝟎, 𝟎, 𝟎)
𝐓𝐡𝐞 𝐜𝐨𝐨𝐫𝐝𝐢𝐧𝐚𝐭𝐞𝐬 𝐨𝐟 𝐚𝐧𝐲 𝐩𝐨𝐢𝐧𝐭 𝐨𝐧 𝐭𝐡𝐞 𝐱 𝐚𝐱𝐢𝐬 𝐢𝐬 𝐢𝐧 𝐭𝐡𝐞 𝐟𝐨𝐫𝐦 𝐨𝐟 (𝐱, 𝟎, 𝟎)
𝐓𝐡𝐞 𝐜𝐨𝐨𝐫𝐝𝐢𝐧𝐚𝐭𝐞𝐬 𝐨𝐟 𝐚𝐧𝐲 𝐩𝐨𝐢𝐧𝐭 𝐨𝐧 𝐭𝐡𝐞 𝐲𝐚𝐱𝐢𝐬 𝐢𝐬 𝐢𝐧 𝐭𝐡𝐞 𝐟𝐨𝐫𝐦 𝐨𝐟 (𝟎, 𝐲, 𝟎)
𝐓𝐡𝐞 𝐜𝐨𝐨𝐫𝐝𝐢𝐧𝐚𝐭𝐞𝐬 𝐨𝐟 𝐚𝐧𝐲 𝐩𝐨𝐢𝐧𝐭 𝐨𝐧 𝐭𝐡𝐞 𝐳 𝐚𝐱𝐢𝐬 𝐢𝐬 𝐢𝐧 𝐭𝐡𝐞 𝐟𝐨𝐫𝐦 𝐨𝐟 (𝟎, 𝟎, 𝐳)
𝐀𝐧𝐲 𝐩𝐨𝐢𝐧𝐭 𝐨𝐧 𝐗𝐘 𝐩𝐥𝐚𝐧𝐞 𝐢𝐬 (𝐱, 𝐲, 𝟎).
𝐀𝐧𝐲 𝐩𝐨𝐢𝐧𝐭 𝐨𝐧 𝐘𝐙 𝐩𝐥𝐚𝐧𝐞 𝐢𝐬 (𝟎, 𝐲, 𝐳).
𝐀𝐧𝐲 𝐩𝐨𝐢𝐧𝐭 𝐨𝐧 𝐙𝐗 𝐩𝐥𝐚𝐧𝐞 𝐢𝐬 (𝐱, 𝟎, 𝐳).

Important Formulas:
Distance formula between two points:
Distance between two points (𝐱𝟏, 𝐲𝟏, 𝐳𝟏) 𝐚𝐧𝐝 𝐐(𝐱𝟐, 𝐲𝟐, 𝐳𝟐) is

|𝐏𝐐| = √(𝐱𝟐 − 𝐱𝟏)𝟐 + (𝐲𝟐 − 𝐲𝟏)𝟐 + (𝐳𝟐 − 𝐳𝟏)𝟐


The distance of the point (𝐱, 𝐲, 𝐳)𝐟𝐫𝐨𝐦 𝐭𝐡𝐞 𝐨𝐫𝐢𝐠𝐢𝐧 𝐎(𝟎, 𝟎, 𝟎)𝐢𝐬

|𝐎𝐏| = √𝐱𝟐 + 𝐲𝟐 + 𝐳𝟐
Distance 𝐨𝐟 (𝐱, 𝐲, 𝐳) from 𝐱 axis is√𝐲𝟐 + 𝐳𝟐

Page 82 of 119
KVSRO/EKM/XI/MATHS

Distance 𝐨𝐟 (𝐱, 𝐲, 𝐳) from 𝐲 axis is√𝐱𝟐 + 𝐳𝟐

Distance 𝐨𝐟(𝐱, 𝐲, 𝐳) from 𝐳axis is√𝐱𝟐 + 𝐲𝟐


𝐑𝐞𝐟𝐥𝐞𝐜𝐭𝐢𝐨𝐧 (𝐢𝐦𝐚𝐠𝐞) 𝐨𝐟 (𝐱, 𝐲, 𝐳) 𝐢𝐧 𝐗𝐘 𝐩𝐥𝐚𝐧𝐞 (𝐱, 𝐲, −𝐳)
𝐑𝐞𝐟𝐥𝐞𝐜𝐭𝐢𝐨𝐧 (𝐢𝐦𝐚𝐠𝐞) 𝐨𝐟 (𝐱, 𝐲, 𝐳) 𝐢𝐧 𝐘𝐙 𝐩𝐥𝐚𝐧𝐞 (−𝐱, 𝐲, 𝐳)
𝐑𝐞𝐟𝐥𝐞𝐜𝐭𝐢𝐨𝐧 (𝐢𝐦𝐚𝐠𝐞) 𝐨𝐟 (𝐱, 𝐲, 𝐳) 𝐢𝐧 𝐙𝐗 𝐩𝐥𝐚𝐧𝐞 (𝐱, −𝐲, 𝐳)

Location of a point P(x,y,z) in three dimensional space.

Mind map:

Page 83 of 119
KVSRO/EKM/XI/MATHS

Sign of Coordinates in different octants:


The sign (+ or -) of the coordinates of a point determines the octant in which the
point lies..
Octants→ I II III IV V VI VII VIII
𝒙 + - - + + - - +
𝒚 + + - - + + - -
𝒛 + + + + - - - -

Distance Formula:
Distance between two points (𝑥1, 𝑦1, 𝑧1) 𝑎𝑛𝑑 𝐵(𝑥2, 𝑦2, 𝑧2) is

|𝐴𝐵| = √(𝑥2 − 𝑥1)2 + (𝑦2 − 𝑦1)2 + (𝑧2 − 𝑧1)2

MCQ Questions:
1.Find the Octant in which the point (-2,5,-3) lies?

(a) Second(b) Sixth (c) Seventh (d)Eighth

2.Distance of the point (3,4,-5) from 𝑿𝒀 plane is

(a) 4 ( b) 3 (c) -5 (d)5

3.Distance of the point (3,6,-8) from x-axis is

(a) 3 ( b) 6 (c) 10 (d) 8

Page 84 of 119
KVSRO/EKM/XI/MATHS

4. The image of the point (3,4,-5) in the 𝒀𝒁 plane is

(a) (3 ,4,5) ( b) (-3,4,-5) (c) (-3 ,4,5) (d ) (3 ,-4,5)

5. The point (0,5,-2) lie in which plane

(a) 𝑿𝒀 𝒑𝒍𝒂𝒏𝒆 (b) 𝒀𝒁 𝒑𝒍𝒂𝒏𝒆 (c) 𝑿𝒁 𝒑𝒍𝒂𝒏𝒆 (d) 𝑵𝒐𝒏𝒆 𝒐𝒇 𝒕𝒉𝒆𝒔𝒆

6. The point (0,0,6) lies on which axis

(a) 𝒙 − 𝒂𝒙𝒊𝒔 (b)− 𝒂𝒙𝒊𝒔 (c) 𝒛 − 𝒂𝒙𝒊𝒔 (d)𝑵𝒐𝒏𝒆 𝒐𝒇 𝒕𝒉𝒆𝒔𝒆

7. The distance between the points (2,0,0) and (-3,0,0) is

(a) 2 (b)3 (c)1 (d) 5

8. Find the distance from the origin to the point (6,-6,-7) is

(a)7 (b) 6 (c) 11 (d) None of these

9. If the distance between the points (a,0,1) and (0,1,2) is √𝟐𝟕,then value of a is
(a) 5 (b) ± 5 (c)-5 (d) None of these

10. What is the length of the foot of perpendicular drawn from the point P(3,4,5) on x-axis

(a) √𝟒𝟏 (b)√𝟑𝟒 (c)√𝟓𝟎 (d)5

SHORT ANSWER QUESTIONS (2 Marks )

1. Prove by distance formula, that the points A(1,-1,3),B(2,-4,5) and

C (5,-13,11)are collinear.

2. Show that the triangle ABC with vertices A(0,4,1),B(2,3,-1) and C(4,5,0) is right angled.

3. Show that the points A (5,-1,1), B (7,-4,7) ,C (1,-6,10) and D (-1,-3,4) are vertices of a Rhombus.

4.Prove by distance formula, that the points A(1,2,3),B(-1,-1,-1) and

C (3,5,7)are collinear.

5.Show that the points A(2,-1,3),B(1,-3,1) and C(0,1,2) are vertices of an isosceles triangle.

6.Show that the points A(2,3,5),B(-4,7,-7),C(-2,1,-10) and D(4,-3,2) are vertices of a rectangle.

7.Find the locus of a point whose each point is equidistant from A(2,3,-4) and B(-1,2,3).

8. Find the locus of a point whose sum of the distances from the points A(2,0,0) and B(-2,0,0) is
10.

Page 85 of 119
KVSRO/EKM/XI/MATHS

9. What are the coordinates of the vertices of a cube whose edge is 2 units, one of whose
vertices coincides with the origin and the three edges passing through the origin, coincides with
the positive direction of the axes through the origin?

10. If the distance between the points (a,2,1) and (1,-1,1) is 5 ,then find the values of a?

CASE STUDY QUESTION

A three-dimensional cartesian Coordinate system is formed by a point called the Origin


(denoted by O) and a basis consisting of three mutually perpendicular vectors. These vectors
define the three coordinate axes :X, Y and Z axes. They are also known as abscissa, ordinate and
applicate axis, respectively. The coordinates of any point in the space is determined by three
real numbers x,y and z.The planes known as coordinate planes divides the space into OCTANTS.
Below shown figure depicts a rectangular parallelopiped. Based on the above information
answer the questions below:

1. If point P represents the coordinate (4,6,5), the coordinates of point N will be?
(a) (4,0,5) (b) (0,6,5) (c)(4,6,0) (d)(0,0,0)

2. In which octant does the point (-4,6,5) lies?

(a) First (b) Second (c)Third (d)Fourth

3.From the figure above, the coordinates of point B will be

(a) (0,6,0) (b) (0,0,5) (c)(0,6,5) (d)(4,6,0)

4.If a point lies on Z-axis then the coordinates of the point will be

(a) (x,y,z) (b) (x,y,0) (c)(0,y,0) (d)(0,0,z)

LONG ANSWER QUESTIONS:

1. Determine the points in XY plane which is equidistant from these points A(2,0,3),B(0,3,2)
and C(0,0,1).

2. Find the coordinate of a point equidistant from the four points O(0,0,0),A(a,0,0),B(0,b,0)
and C(0,0,c).

Page 86 of 119
KVSRO/EKM/XI/MATHS

3. If A and B are the points (-2,2,3) and (-1,4,-3) respectively, then find the locus of P such that
3|𝑃𝐴| = 2|𝑃𝐵|

***************************

ANSWERS:

MCQ Questions:

1.(b) Sixth 2.(d) 5 3. (c) 10 4. (b) (-3,4,-5) 5.(b) YZ plane

6. (c) z-axis 7.(d) 5 8.(c)11 9.(b) ±5 10. (a) √𝟒𝟏


SHORT ANSWER QUESTIONS (2 Marks )

1.AB = √𝟏𝟒 , BC = 3√𝟏𝟒& AC = 4√𝟏𝟒 .Hence AC= AB +BC .Hence the points are collinear.

2.AB =3,BC =3 & AC = √𝟏𝟖.Length of sides satisfy the condition of Pythagoras

Theorem.
3.Prove that all sides are equal and diagonals are not equal.

4.AB = √𝟐𝟗 , BC = 2= √𝟐𝟗& AC = = √𝟐𝟗.Hence BC= AB +AC .Hence the points are collinear.
5.Prove that the length of any two sides are equal.

6.Prove that the length of opposite sides are equal and diagonals are also equal.

7.6x+2y-14z+15+0.

8.21𝒙𝟐+25𝒚𝟐+25𝒛𝟐= 525.

9.Coordinates of the vertices of a cube whose edge is 2 units are


(2,0,0),2,2,0),(0,2,0),(0,2,2),(0,0,2),(2,0,2),(0,0,0) and (2,2,2)

10.a=5 or -3.

CASE STUDY QUESTION

1.(b) (0,6,5) 2.(b)Second 3.(a) (0,6,0) 4.(d) (0,0,z)

LONG ANSWER QUESTIONS:


𝒂𝒃𝒄
1. Ans:(3,2,0) 2.Ans: P( , , )
𝟐 𝟐 𝟐
3. Ans: 𝟓𝒙𝟐 + 𝟓𝒚𝟐 + 𝟓𝒛𝟐 + 𝟐𝟖𝒙 − 𝟒𝒚 − 𝟑𝟎𝒛 + 𝟒𝟗 = 𝟎.

Page 87 of 119
KVSRO/EKM/XI/MATHS

Chapter 12
LIMITS AND DERIVATIVES
Important Points to Remember
1. The limit of a function is the common value of the left and right hand limits, if they
coincide.
𝐿𝑖𝑚 − 𝑓(𝑥) = 𝐿𝑖𝑚 + 𝑓(𝑥) = 𝐿𝑖𝑚
𝑓(𝑥)
𝑥→𝑎 𝑥→𝑎 𝑥→𝑎
2. For function 𝑓 and 𝑔 the following holds
𝐿𝑖𝑚 𝐿𝑖𝑚
i) 𝐿𝑖𝑚 [𝑓(𝑥) ± 𝑔(𝑥)] = 𝑓(𝑥) ± 𝑔(𝑥)
𝑥→𝑎 𝑥→𝑎 𝑥→𝑎
𝐿𝑖𝑚 𝐿𝑖𝑚
ii) 𝐿𝑖𝑚 [𝑓(𝑥) × 𝑔(𝑥)] = 𝑓(𝑥) × 𝑔(𝑥)
𝑥→𝑎 𝑥→𝑎 𝑥→𝑎
𝐿ⅈ𝑚
𝑓(𝑥)
𝐿𝑖𝑚 [𝑓(𝑥) ] = 𝑥→𝑎 provided 𝐿𝑖𝑚
iii) iii) 𝐿ⅈ𝑚 (𝑥) ≠ 0
𝑥 → 𝑎 𝑔(𝑥) 𝑔(𝑥) 𝑥→𝑎
𝑥→𝑎

3. Standard Limits

i) 𝐿𝑖𝑚
𝑥𝑛−𝑎
= 𝑛𝑎𝑛−1
𝑥 → 𝑎 𝑥−𝑎
ii) 𝐿𝑖𝑚
𝑆ⅈ𝑛𝑥
=1
𝑥→0 𝑥
𝐿𝑖𝑚 1−𝑐𝑜𝑠𝑥
iii) =0
𝑥→0 𝑥

4. Derivative of a function at any point 𝑥 is defined by


𝑑 𝐿𝑖𝑚
𝑓1(𝑥) = (𝑓(𝑥)) = 𝑓(𝑥+𝑏)−𝑓(𝑥)
𝑑𝑥 ℎ→0 ℎ

5. For the function 𝑢 and 𝑣 the following holds


i) (𝑢 ± 𝑣)1 = 𝑢1 ± 𝑣1
ii) (𝑢. 𝑣)1 = 𝑢1𝑣 + 𝑢𝑣1
𝑢 1 𝑢1𝑣−𝑢𝑣 1
iii) ( ) = provided all the defined...
𝑣 𝑣2

Standard Derivatives
𝑑 𝑥
(𝑥 ) = 𝑛𝑥𝑛−1
𝑑𝑥
𝑑
(𝑠𝑖𝑛𝑥) = 𝑐𝑜𝑠𝑥
𝑑𝑥
𝑑
(𝑐𝑜𝑠𝑥) = −𝑠𝑖𝑛𝑥
𝑑𝑥

Page 88 of 119
KVSRO/EKM/XI/MATHS

𝑑
(𝑡𝑎𝑛𝑥) = 𝑠𝑒𝑐2𝑥
𝑑𝑥
𝑑
(𝑐𝑜𝑠𝑒𝑐𝑥) = −𝑐𝑜𝑠𝑒𝑐𝑥 𝑐𝑜𝑡𝑥
𝑑𝑥
𝑑
(sec 𝑥) = 𝑠𝑒𝑐𝑥 𝑡𝑎𝑛𝑥
𝑑𝑥
𝑑
(𝑐𝑜𝑡 𝑥) = −𝑐𝑜𝑠𝑒𝑐2𝑥
𝑑𝑥

MCQS (1 mark)
𝐿𝑖𝑚 √2𝑐𝑜𝑠𝑥−1
1. 𝑥 → 4 𝑐𝑜𝑡𝑥−1 is equal to
𝜋

a) 1 b) 1 c) 1 d) 1
√2 2 2√2

𝐿𝑖𝑚 1−𝑐𝑜𝑠2𝑥
2.
𝑥
is
𝑥→0
a) 0 b) 1 c) 2 d) 4
𝑠𝑖𝑛4𝑥
3.
𝐿𝑖𝑚 equal to
𝑥 → 0 𝑠𝑖𝑛2𝑥
a) 1 b) 4 c) 2 d) 8
2
𝑥 𝑥2 𝑥3
4. If 𝑦 = + + + ⋯,then 𝑑𝑦 is
11 21 3 𝑑𝑥
a) 𝑦 + 1 b) 𝑦 − 1 c) 𝑦 d) 𝑦2
𝑠ⅈ𝑛𝑥+𝑐𝑜𝑠𝑥
5. If 𝑦 = then 𝑑𝑦 at 𝑥 = 0 is
𝑠ⅈ𝑛𝑥−𝑐𝑜𝑠𝑥 𝑑𝑥

a) −2 b) 0 c) 1 d) 1
2

Short Answer Type Questions (2 marks)


1 1
6. Evaluvate 𝐿𝑖𝑚 (𝑥+2)3−23
𝑥→0 𝑥
𝐿𝑖𝑚 1−𝑐𝑜𝑠𝑥
7. Evaluvate
𝑥 → 0 𝑥2
𝐿𝑖𝑚 𝑥𝑛−2𝑛
8. If = 448, find the value of 𝑛.
𝑥→2 𝑥−2
(1+𝑥)4−1
9. Evaluvate 𝑙𝑖𝑚
𝑥
𝐿𝑖𝑚 1−𝑐𝑜𝑠4𝑥
10. Find
𝑥 → 0 1−𝑐𝑜𝑠5𝑥

Page 89 of 119
KVSRO/EKM/XI/MATHS

𝑑𝑦
11. If 𝑦 = 𝑥2𝑠𝑖𝑛𝑥 + 𝑐𝑜𝑠2𝑥, 𝑓𝑖𝑛𝑑
𝑑𝑥
1 3
12. Differentiate (𝑥 + ) with respect to 𝑥.
𝑥
13. Differentiate 𝑦 = 𝑠𝑖𝑛3𝑥 𝑐𝑜𝑠3𝑥 with respect to 𝑥.
𝑥
14. If 𝑦 = then find 𝑑𝑦
1+𝑡𝑎𝑛𝑥 𝑑𝑥
𝑥2𝑛
15. Differentiate with respect to 𝑥.
𝑡𝑎𝑛𝑥

Long answer type Questions (5 marks)


𝜋
𝑘𝑐𝑜𝑠𝑥 𝑖𝑓 𝑥≠ 𝐿𝑖𝑚
2 𝜋
16. If (𝑥) = {𝜋−2𝑥 and if 𝑓
𝑖 𝑥→𝜋 𝜋 (𝑥) = 𝑓 ( ), find the value of 𝑘.
3 𝑖𝑓 𝑥 = 2 2
2

3𝑎𝑥 + 𝑏, 𝑥>1
17. If (𝑥) = { 11 𝑖𝑓 𝑥 = 1
5𝑎𝑥 − 2𝑏 𝑖𝑓 𝑥 < 1
𝐿𝑖𝑚 (𝑥) = 𝑓(1), find the values of 𝑎 and 𝑏.
If
𝑥→1
18. Differentiate 𝑥2𝑠𝑖𝑛𝑥 using first principal.
𝐿𝑖𝑚 𝑠ⅈ𝑛𝑥−𝑐𝑜𝑠𝑥
19. Find 𝑥 → 𝜋 𝑥−
𝜋
4 4

𝑠𝑖𝑛𝑥−𝑥 𝑐𝑜𝑠 𝑑𝑦
20. If 𝑦 = , find
𝑥 𝑠𝑖𝑛𝑥 + 𝑐𝑜𝑠𝑥 𝑑𝑥

Case Study
21. If (𝑥) is a polynomial function if it satisfies (𝑥) = 𝑎𝑛𝑥𝑛 + 𝑎𝑛−1𝑥𝑛−1 + ⋯ + 𝑎1𝑥 +
𝑎0(𝑎𝑛 ≠ 0) where 𝑛 is a non negative integer and 𝑎1, 𝑎2 … 𝑎𝑛are constants. Since
polynomial can be treated as sum of functions, we can use the property “limit of the
sum is equal to sum of the limits” to determine limits. Based on the above information,
answer the following questions.
𝐿𝑖𝑚 𝑥3+8
i) The value of is
𝑥 → 2 𝑥2−1
16
a) 0 b) 8 c) 3 d) 𝑛𝑜𝑡 𝑑𝑒𝑓𝑖𝑛𝑒𝑑
3
𝐿𝑖𝑚 𝑥2−9
ii) The value of is
𝑥 → 0 𝑥2+9𝑥−6𝑥2
a) 0 b) 3 c) 6 d) 𝑛𝑜𝑡 𝑑𝑒𝑓𝑖𝑛𝑒𝑑

Page 90 of 119
KVSRO/EKM/XI/MATHS

𝑥2+1
iii) The derivative of with respect to 𝑥 is
𝑥+2
𝑥2+4 𝑥2−4𝑥+1 𝑥2+4𝑥−1 𝑥2−4𝑥−1
a) b) c) d)
(𝑥+2)2 𝑥2−4𝑥+4 𝑥2+4𝑥+4 𝑥2−4𝑥+4

Answers
1 1 1 −2
1) 1 2) 0 3) 2 4) 𝑦 5) −2 6) (𝑥 + 2)3−1 = (𝑥 + 3) 3
2 3 3

7) 1 8) 𝑛 = 7 9) 4 10) 16 11) 𝑥2𝑐𝑜𝑠𝑥 + 2𝑥 𝑠𝑖𝑛𝑥 − 2 𝑠𝑖𝑛2𝑥


2 25

1 2 1
12) 3 (𝑥 + ) . (𝑥 + ) 13) −3𝑠𝑖𝑛4 𝑥𝑐𝑜𝑠 2𝑥 + 3𝑐𝑜𝑠 2𝑥 𝑠𝑖𝑛2𝑥
𝑥 𝑥2

2𝑛𝑥2𝑛−1𝑡𝑎𝑛𝑥−𝑥2𝑛𝑠𝑒𝑐2𝑥 1+𝑡𝑎𝑛𝑥−𝑥𝑠𝑒𝑐2𝑥
14) 15)
𝑡𝑎𝑛2𝑥 (1+𝑡𝑎𝑛𝑥)2
𝜋 𝜋
16) = 6, (𝐻𝑖𝑛𝑡 𝐴𝑡 𝑥 = , 𝐿𝐻𝐿 = 𝑅𝐻𝐿 = 𝑓 ( )]
2 2

17) 𝑎 = 3, 𝑏 = 2; 𝐴𝑡 𝑥 = 1
𝐿𝐻𝐿 = 𝑅𝐻𝐿 = (1)

18) 𝑑𝑦 = 𝑥2𝑐𝑜𝑠𝑥 + 2𝑥 𝑠𝑖𝑛𝑥


𝑑𝑥

𝐿𝑖𝑚 𝑠ⅈ𝑛𝑥−𝑐𝑜𝑠𝑥 𝐿𝑖𝑚 𝑠𝑖𝑛𝑥−𝑐𝑜𝑠𝑥 𝐿𝑖𝑚 𝑆ⅈ𝑛𝑥 1 −𝐶𝑜𝑠𝑥 1


𝜋 √2 √2 √2 𝜋 √2
19) 𝑥 → 4𝑥−𝜋 = √2 𝑥 → 𝜋 4𝑥−𝜋 = 4 𝑥 →𝜋 𝑥−
4 4 4 4

𝐿𝑖𝑚 𝜋 𝜋
√2 𝑆𝑖𝑛𝑥 𝑐𝑜𝑠 − 𝐶𝑜𝑠𝑥 𝑠𝑖𝑛𝑥
= 𝜋 4 𝜋 4
4 𝑥→ 𝑥−
4 4
𝐿𝑖𝑚 𝜋
2 2
√2
= 4 𝑥 → 𝜋 𝑆ⅈ𝑛𝑥−
(𝑥−
𝜋 4)
= √ ×1=√
4 4
4 4

𝑥2 1 𝑣𝑢1−𝑢𝑣1
( 𝐻𝑖𝑛𝑡 𝑢𝑠𝑒 ( ) = )
20) 𝑣 𝑣2
(𝑥 𝑠ⅈ𝑛𝑥+𝑐𝑜𝑠𝑥)2
2+𝑢𝑥−1
16 ii) not defined iii) 3 iv) 𝑥
21) i)
3 𝑥2+4𝑥+4

Page 91 of 119
KVSRO/EKM/XI/MATHS

Chapter 13
STATISTICS
IMPORTANT FACTS AND FORMULAS

1. Class Limits:
Each class in a frequency distribution has a lower-class limit and an upper-class limit: Lower
class limit: The smallest data value that can belong to a class. Upper class limit: The largest
data value that can belong to a class.
2. Type of class intervals
There are two type of class intervals namely Exclusive class intervals and Inclusive class
intervals
Exclusive class intervals in which upper limit excluded and lower limit included
Example
0-10,10-20,20-30 in the class 10-20 the observation 20 excluded and 10 included
Inclusive class intervals both upper and lower limits are included
Example
1-8,9-16,17- 24, In the class 9-16 both lower limit 9 and upper limit 16 are included
3. True lower limits and upper limits of the class 10-20 in the exclusive class intervals 0-10,10-
20,20-30 is 10 and 20 respectively
4. True lower limits and upper limits of the class 9--16 in the inclusive class intervals 1-8,9-
8+9 16+17
16,17- 24 is 2
= 8.5 and 2
= 16.5 respectively
5. Class Size or class width = true upper limit- true lower limit
6. Example
Class size of class 10-20 in the exclusive form 0-10,10-20,20-30 is 20-10=10
Class size of class 9-16 in the inclusive form1-8,9-16,17- 24 is 16.5-8.5=8
𝑢𝑝𝑝𝑒𝑟 𝑙ⅈ𝑚ⅈ𝑡+𝑙𝑜𝑤𝑒𝑟 𝑙ⅈ𝑚ⅈ𝑡 𝑡𝑟𝑢𝑒 𝑢𝑝𝑝𝑒𝑟 𝑙ⅈ𝑚ⅈ𝑡+𝑡𝑟𝑢𝑒 𝑙𝑜𝑤𝑒𝑟 𝑙ⅈ𝑚ⅈ𝑡
7. Mid value of a class = =
2 2
8. Mean of the observations 𝑥1,𝑥2 ,𝑥3……𝑥𝑛 is denoted as 𝑥̅,
𝒙 +𝒙 𝒙 …+𝒙 𝟏
̅= 𝟏 𝟐+ 𝟑 𝒏 = ∑𝒏
𝒙 𝒙 = 𝒔𝒖𝒎 𝒐𝒇 𝒕𝒉𝒆 𝒐𝒃𝒔𝒆𝒓𝒗𝒂𝒕𝒊𝒐𝒏
𝒏 𝒏 𝒊=𝟏 𝒊 𝒏𝒖𝒎𝒃𝒆𝒓 𝒐𝒇 𝒐𝒃𝒔𝒆𝒓𝒗𝒂𝒕𝒊𝒐𝒏𝒔
9. Sum of the observations =∑𝒏𝒊=𝟏 𝒙𝒊 =n𝒙̅
10. Mean of a discrete and continuous frequency distribution
𝟏 𝒏
̅ = 𝑵∑
𝒙 𝒇𝒊𝒙𝒊 where N=∑𝒏 𝒇
𝒊=𝟏 𝒊=𝟏 𝒊

In the case discreate frequency distribution 𝑥ⅈ is the value of the variable in 𝑖𝑡ℎ class
In the case of continuous frequency distribution.𝑥ⅈmid value 𝑖𝑡ℎ class
𝑓ⅈ is the frequency of 𝑥ⅈ
11. Short cut method for Mean
𝑥 −𝐴
If 𝑑 = 𝑖 then 𝑥̅ =A + c𝑑̅ where A is assumed mean and h common divisors of𝑥 − 𝐴 in the
ⅈ ℎ ⅈ
case of raw data otherwise h is the uniform class size

Page 92 of 119
KVSRO/EKM/XI/MATHS

12. The median of raw data is the number which divides the observations when arranged in an
order (ascending or descending) in two equal parts.
13. Method of finding median from raw data and discreate frequency table
Take the following steps to find the median of raw data and discreate frequency table
Step I: Arrange the raw data in ascending or descending order.
Step II: Let the number of observations in the data is n.
𝒏+𝟏
(i) If n is odd then ( )𝒉observation is the median.
𝟐
𝟏 𝒏 𝒏
(ii) If n is even {( ) 𝒕 observation+( +1) thobservation}is the median
𝒉
𝟐 𝟐 𝟐
14. Method of finding median from raw continuous frequency table
Write cumulative frequency table with true class limits
N= Sum of frequencies
𝑁
Median class: class interval whose cumulative frequency is just greater or equal to
2
C- Cumulative frequency of the class just preceding to median class

l -true lower limit of the median class


h-size of median class

f – frequency of median class


𝑁
−𝐶
Median = l + ( 2 )h
𝑓
15. Measures of dispersion
Dispersion measures the degree of scatteredness of the variable about a central value
16. Different measures of dispersion: Range, Mean deviation, Standard deviation
17. Range =largest observation – smallest observation
18. Mean deviation about Mean, M.D(𝒙 ̅)
If 𝑥1 ,2 , 𝑥3 ……𝑥𝑛 are “n “observation with mean 𝒙
̅ then and median M
𝟏
̅)= ∑𝒏
Mean deviation about Mean, M.D(𝒙 |𝒙 − 𝒙
̅|
𝒏 𝒊=𝟏 𝒊

𝟏
Mean deviation about Median, M.D(M)= ∑𝒏 |𝒙 − 𝑴|
𝒏 𝒊=𝟏 𝒊

In case of discrete and continuous series


𝟏
̅)= ∑𝒏
M.D(𝒙 𝒇 |𝒙 − 𝒙
̅| where N= ∑𝒏 𝒇
𝑵 𝒊=𝟏 𝒊 𝒊 𝒊=𝟏 𝒊

𝟏
4M.D(M)= ∑𝒏 𝒇 |𝒙 − 𝑴| where N= ∑𝒏 𝒇
𝑵 𝒊=𝟏 𝒊 𝒊 𝒊=𝟏 𝒊

In the case discreate frequency distribution 𝑥ⅈ is the value of the variable in 𝑖𝑡ℎ class
In the case of continuous frequency distribution.𝑥ⅈmid value of𝑖𝑡ℎ class
𝑓ⅈ is the frequency of 𝑥ⅈ

Page 93 of 119
KVSRO/EKM/XI/MATHS

19. Variance: Mean of squared deviations


If 𝑥1 ,2 , 𝑥3 ……𝑥𝑛 are “n “observation with mean 𝒙
̅
𝟏
Variance of X, V(X) = 𝝈𝟐= ∑𝒏 (𝒙 − 𝒙 ̅)𝟐
𝒏 𝒊=𝟏 𝒊
𝒏 ∑𝒏 𝒙𝒊𝟐−(∑𝒏 𝒙𝒊)𝟐
Formula for Calculation 𝝈𝟐= 𝟏 𝒏𝟐 𝟏
In case of discrete and continuous series
𝟏
Variance of X, V(X) = 𝝈𝟐= ∑𝒏 𝒇 (𝒙 − 𝒙 ̅)𝟐 where N= ∑𝒏 𝒇
𝑵 𝒊=𝟏 𝒊 𝒊 𝒊=𝟏 𝒊
𝑵 ∑𝒏 𝒇𝒊𝒙𝒊𝟐−(∑𝒏 𝒇𝒊𝒙𝒊)𝟐
𝟏
Formula for Calculation 𝝈𝟐= 𝟏
𝑵𝟐
Standard deviation of X = S.D(X)=𝜎=√𝑽(𝑿)
20. Short cut method
𝒉𝟐 𝒉
𝝈 𝟐= [𝑵 ∑𝒏 𝒇 𝒚 𝟐 − (∑𝒏 𝒇 𝒚 )𝟐] and 𝜎 = √𝑵 ∑𝒏 𝒇 𝒚 𝟐 − (∑𝒏 𝒇 𝒚 )𝟐] where 𝒚 𝒙𝒊−𝑨
𝑵𝟐 𝟏 𝒊 𝒊 𝟏 𝒊 𝒊 𝑵 𝟏 𝒊 𝒊 𝟏 𝒊 𝒊 𝒊 = 𝒉
In the case discreate frequency distribution 𝑥ⅈ is the value of the variable in 𝑖𝑡ℎ class
In the case of continuous frequency distribution.𝑥ⅈmid value of𝑖𝑡ℎ class
𝑓ⅈ is the frequency of 𝑥ⅈ

MIND MAP

Page 94 of 119
KVSRO/EKM/XI/MATHS

MCQ (1 MARK)

1) Which of the following is not a measure of dispersion?


(a) S.D b) Range c) Variance d) Median
2) Mean deviation of the data 3, 10,10,4,7,10,5 from the mean is
(a) 2 b) 2.57 c) 3 d) 3.75
3) If the mean of 𝑥1, 𝑥2, … … . . 𝑥𝑛 is 𝑥̅, then mean of n observations 2𝑥1 + 3 , 2𝑥2+3 ,…,2𝑥𝑛
(a)3 𝑥̅ +2 b) 2 𝑥̅ +3 c) 2 𝑥̅ d) 3 𝑥̅
4) S.D of first n natural numbers is
𝑛+1 √n2−1 2 −1
(a) b) c) √n None of these
2 12 d) 12
5) If the variance of the numbers 2, 4,5,6,8,17 is 23.33, then the variance of 4,8,10,12,16,3.4 will be
23.33 b) 46.66 c) 93.32 d ) none of these

Short answer questions (2 Marks)

6) Find the mean deviation about the mean for the data 4, 7, 8, 9, 10 ,12, 13, 17
7) Find the mean deviation about the median for the data 36, 72, 46, 42, 60, 45, 53, 46
8) Find the mean deviation about mean for the data
𝑥ⅈ 10 30 50 70 90
𝑓ⅈ 4 24 28 16 8
9) Find the mean deviation about median for the data
𝑥ⅈ 5 7 9 10 12 15
𝑓ⅈ 8 6 2 2 2 6
10) Find the mean andvariance of 6,7,10,12,13,4, 8, 12
11) Find the mean and variance of first n natural numbers
12) Find the mean and variance of the data
𝑥ⅈ 6 10 14 18 24 28 30

𝑓ⅈ 2 4 7 12 8 4 3
13) Find the standard deviation of the data
𝑥ⅈ 2 3 4 5 6 7
𝑓ⅈ 4 9 16 14 11 6
14) The mean and standard deviation of six observations are 8 and 4 respectively. If each observation
is multiplied by 3, find the new mean and new standard deviation of the resulting observations
15) Given that 𝑥̅ is the mean and 𝝈𝟐 is the variance of n observations 𝑥1, 𝑥2, ..........𝑥𝑛 Find the
mean and variance of the observations a 𝑥1, 𝑎 𝑥2, … … . 𝑎𝑥𝑛 where a ≠0
Case study questions

16) The marks obtained by 20 students in a test were


13, 17, 11, 5,18,16,11,14,13,12,18,11,9,6,8,17,21,22,7,6
Calculate the following
i) The mean marks per student
(ii) The mean marks per student when marks of each student are increased by 5
(iii) The mean marks per student when 2 marks are deducted from the marks of each student
(iv) The mean marks when the marks of each student are doubled
17) A survey was made in a class to analyse the performance of students in a particular subject.

Page 95 of 119
KVSRO/EKM/XI/MATHS

While doing the analysis the mean and standard deviation of marks of 20 students are found to be
10 and 2 respectively. On rechecking it was found that an observation 8 was incorrect. Calculate
the correct mean and standard deviation in each of the following cases
i)If the wrong item is omitted
ii)If it is replaced by 12
LONG ANSWERS
18) Find the mean deviation about the mean for the following data
Height in cms : 95-105 105-115 115-125 125-135 135-145 145-155

Numberof boys: 9 13 26 30 12 10

19) Find the mean deviation about median for the following data
Marks: 0-10 10-20 20-30 30-40 40-50 50-60
No of students: 6 8 14 16 4 2
20) Find the mean, variance and standard deviation of the following data using short cut Method
Height in cms. 70-75 75-80 80-85 85-90 90-95 95-100 100-105 105-110 110-115

No. of children 3 4 7 7 15 9 6 6 3

21) The mean and variance of 7observations are 8 and 16 respectively.If five of the observations are
2,4,10,12,14, find the remaining two observations

ANSWERS
𝑛+1
1) d 2) b 3) b 4) c 5) c (6) 3 7) 7 8) 16 9) 3.23 10) 9, 9.25 11) 2
𝑛2 −1
, 12) 19, 43.4 13) 1.38 14) 24, 12 15) a 𝑥̅ , 𝑎2𝝈𝟐 16) I) 12.75 ii) 17.75 iii)
12

10.75 iv) 25.5 17) i) 10.1, 1.997 ii) 10.2, 1.9918)11.28 19) 10.34 20) 93, 105.52, 10.27 21)
6,8

Page 96 of 119
KVSRO/EKM/XI/MATHS

Chapter 14
PROBABILITY
SUMMARY

Sample space: The set of all possible outcomes is called a sample space

Sample points: Elements of sample space are called sample points

Event: A subset of the sample space is called an event

Impossible event:Impossible event is an empty set

Sure event:Sure event is the whole sample space

Complementary event or ‘not event’:The set A′ or S – A is the complementary event of A

Event A or B: The set A 𝖴 B

Event A and B: The set A ∩ B

Event A and not B: The setA– B

Mutually exclusive event: A and B are mutually exclusive event if A ∩ B = φ

Exhaustive and mutually exclusive events: Events E1,E2... En are mutually exclusive and exhaustive
events if E1𝖴 E2𝖴 ...𝖴 En = S and Ei∩ Ej = φ for all i≠ j

Probability: Number P (ωi ) associated with sample point ωi such that

(i) 0 ≤ P (ωi ) ≤ 1

(ii) ∑P(ωi)=1 for all ωi ∈ S

(iii) P(A) = ∑P(ωi)for all ωi∈A. The number P (ωi ) is called probability of the outcome ωi.

Equally likely outcomes: All outcomes with equal probability is called equally likely outcome.

Probability of an event: For a finite sample space with equally likely outcomes Probability of an event P
𝑛(𝐴)
(A) = 𝑛(𝑆) , where n (A) = number of elements in the set A, n(S) = number of elements in the set S.

If A and B are any two events, then P (A or B) = P (A) + P (B) – P (A and B) equivalently,

P (A 𝖴 B) = P (A) + P (B) –P (A ∩ B)

If A and B are mutually exclusive, then P (A or B) = P (A) + P(B)

If A is any event, then P (not A) = 1 – P (A)

Page 97 of 119
KVSRO/EKM/XI/MATHS

MULTIPLE CHOICE QUESTIONS

1. Two dice are thrown together. The probability of obtaining total score of seven is

a) 5/36 b)1/6 c)7/36 d)2/9

2. A and B are two events such that P (A) = 0.25 and P(B) = 0.50. The probability of both happening
together is 0.14. The probability of both A and B not happening is

(a) 0.39(b) 0.25(c) 0.11(d) none of these

3. A pack of cards contains 4 aces, 4 kings, 4 queens and 4 jacks. Two cards are drawn at random. The
probability that at least one of them is an ace is

(a) 1/5(b) 3/16 (c)9/20 (d) 1/9

4. Three coins are tossed once. Find the probability of getting exactly one tail is

(a) 7/8(b) 3/8 (c)1/8 (d) 1/2

5. One card is drawn from a pack of 52 cards. The probability that it is the card of a king or spade is

(a) 1/26(b) 3/26 (c)4/13 (d) 3/13

ANSWER

1. b 2. a 3. c 4.b 5c
SHORT ANSWER QUESTIONS

1. A coin is tossed three times, consider the following events. A: ‘No head appears’, B: ‘Exactly one
head appears’ and C: ‘At least two heads appear’. Do they form a set of mutually exclusive and
exhaustive events?
2. Two students Anil and Ashima appeared in an examination. The probability that Anil will qualify
the examination is 0.05 and that Ashima will qualify the examination is 0.10. The probability that
both will qualify the examination is 0.02. Find the probability that both Anil and Ashima will not
qualify the examination
3. A coin is tossed twice, what is the probability that atleast one tail occurs?
4. Given P (A) = 3/5 and P (B) = 1/5. Find P(A or B), if Aand B are mutually exclusive events.
5. A die has two faces each with number ‘1’, three faces each with number ‘2’ and one face with
number ‘3’. If die is rolled once, determine
(i) P(2)
(ii) P(1 or 3)
6. One number is chosen from numbers 1 to 100. Find the probability that it is divisible by 4 or 6?
7. A and B are mutually exclusive events such that P(A)= 0.35 P(B)=0.45, find
(i) P(Aꓵ𝑩 ̅)
(ii) P(𝑨 ̅)
̅ ꓵ𝑩
8. A die is thrown twice. What is the probability that atleast one of the two throws comes up with
the number 4?

Page 98 of 119
KVSRO/EKM/XI/MATHS

9. A drawer contains 50 bolts and 150 nuts. Half of the bolts and half of the nuts are rusted. If one
item is chosen at random, what is the probability that it is rusted or a bolt?
10. In a single throw of two dice,find the probability that neither a doublet nor a total of 9 will
appear?

HINT AND SOLUTION

1. A 𝖴 B 𝖴 C = {TTT, HTT, THT, TTH, HHT, HTH, THH, HHH} = S

Therefore, A, B and C are exhaustive events.

Also, A ∩ B = φ, A ∩ C = φ and B ∩ C = φ

Therefore, the events are pair-wise disjoint, i.e., they are mutually exclusive. Hence, A, B and C form a set
of mutually exclusive and exhaustive events.

2. 0.87

3. P{ TH,HT,TT}= 3/4

4. P(AꓴB)= P(A)+ P(B)=4/5

5. (i) ½ (ii) ½

6. 33/100

7. (i) P(Aꓵ̅𝑩̅) =P(A)= 0.35

̅ ꓵ̅𝑩̅)=P(AꓴB)’=1-P(AꓴB)=1-(P(A)+P(B))=1-(0.35+0.45)=0.2
(ii) P(𝑨

8.P{1,4),(2,4),(3,4),(4,4),(5,4),(6,4),(4,1),(4,2),(4,3),(4,5),(4,6)}=11/36

9. Required probability=100/200+50/200-25/200=5/8

10. Let A=Getting a doublet,B=Getting a total of 9.

Required probability=1-P (AꓴB) =1-(6//36+4/36)=13/18

CASE-STUDY BASED QUESTIONS

Q1. A urn contains twenty white slips of paper numbered from 1 through 20, ten red slips of paper
numbered from 1 through 10, forty yellow slips of paper numbered from 1 through 40 and ten blue slips of
paper numbered from 1 through 10. These 80 slips of paper are thoroughly shuffled so that each slip has
the equal chance of being drawn. A slip is drawn at random from the urn.Based on the above information,
answer the following questions

(1) The probability that slip drawn is blue or white is

a) 1/4 b) 3/8 c) 1/2 d) 5/8

Page 99 of 119
KVSRO/EKM/XI/MATHS

(2) The probability that slip drawn is numbered 1,2,3,4 or 5 is

a) 1/4 b) 3/8 c) 1/2 d) 5/8

(3) The probability that slip drawn is red or yellow and numbered 1, 2, 3 or 4 is

a) 1/5 b) 1/10 c) 2/5 d) 3/10

(4) The probability that slip drawn is numbered 5,15, 25or 35 is

a) 3/10 b) 1/5 c) 1/10 d) 2/5

ANSWER

1. b 2. a 3. b 4. c

Q2. In a class of 60 students 30 opted for NCC, 32 opted for NSS and 24 opted for both NCC and NSS.

If one of these students is selected at random, find the probability that:

1. The student opted for NCC or NSS

a) 19/30 b) 11/30 c) 2/15 d) 1/10

2. The student has opted neither NCC nor NSS

a) 19/30 b) 11/30 c) 2/15 d) 1/10

3.The student has opted NSS but not NCC

a) 19/30 b) 11/30 c) 2/15 d) 1/10

4. The student has opted NCC but not NSS

a) 19/30 b) 11/30 c) 2/15 d) 1/10

Page 100 of 119


KVSRO/EKM/XI/MATHS

ANSWER

1. a 2. b 3. c 4. d

LONG ANSWER QUESTIONS

1.A bag contains 9 discs of which 4 are red, 3 are blue and 2 are yellow. The discs are similar in shape and
size. A disc is drawn at random from the bag. Calculate the probability that it will be

(i) red

(ii) yellow

(iii) blue

(iii) Either red or blue.


2.A committee of two persons is selected from two men and two women. What is the probability that
the committee will have (a) no man? (b) One man? (c) Two men?

3. In a relay race there are five teams A, B, C, D and E.

(a) What is the probability that A, B and C finish first, second and third, respectively.

(b) What is the probability that A, B and C are first three to finish (in any order) (Assume that all finishing
orders are equally likely)

4. If a person visits his dentist, suppose the probability that he will have his teeth cleaned is 0.48, the
probability that he will have cavity filled is 0.25, probability that he will have a tooth extracted is 0.20,
the probability that he will have a teeth cleaned and cavity filled is 0.09, the probability that he will have
his teeth cleaned and a tooth extracted is 0.12, the probability that he will have a cavity filled and tooth
extracted is 0.07, and the probability that he will have his teeth cleaned, cavity filled, and tooth
extracted is 0.03 What is the probability that a person visiting his dentist will have at least one of these
things done to him?

5. An integer is chosen at random from the numbers ranging from 1 to 50.What is the probability that the
integer chosen is multiple of 2 or 3 or 10?

HINT AND SOLUTION


1.
(i) The number of red discs = 4, i.e., n (A) = 4 Hence P(A) = 4/9
(ii) The number of yellow discs = 2, i.e., n (B) = 2 Therefore, P(B) = 2/9
(iii) The number of blue discs = 3, i.e., n(C) =3/9
(iv) The event ‘either red or blue’ may be described by the set ‘A or C’ Since, A and C
are mutually exclusive events, we have P(A or C) = P (A 𝖴 C) = P(A) + P(C) = 7/9
2.

(a) P( no man)= 2C2/4C2=1/6

(b) P(One man)=(2C1x2C1)/4C2=2/3

Page 101 of 119


KVSRO/EKM/XI/MATHS

(c) P(Two men)= 2C2/4C2=1/6

3. (a) No of sample points=5P3=60

Required Probability=1/60

(b) Required probability=3! /60=1/10

4. Consider the following events:

C= the person will have his teeth cleaned,

F = the person will have cavity filled

E= the person will have a tooth extracted

It is given that P(C)=0.48, P(F)=0.25, P (E) =0.20,

P(CꓵF)=0.09, P (CꓵE) =0.12, P (EꓵF)=0.07 and P (CꓵFꓵE)=0.03.

Required probability = P(CUFUE)= P(C)+P(F)+P(E)-P(CꓵF)-P (FꓵE)-P(CꓵE)+P(CꓵFꓵE)=-0.48+0.25 +0.20-


0.09-0.12-0.07 +0.03 =0.68

5. Let A=Getting a multiple of 2

B= Getting a multiple of 3

C= Getting a multiple of 10

Required probability=P(AꓴBꓴC)=25/50+16/50+5/50-8/50-1/50-5/50+1/50=33/50

==================================================================================

Page 102 of 119


KVSRO/EKM/XI/MATHS

Sample Question Paper -1


(Session 2022-23)
Class XI Sub: Mathematics
Time Allowed: 3 Hours Maximum Marks: 80
General Instructions :
1. This Question paper contains - five sections A, B, C, D and E. Each section is compulsory.
However, there are internal choices in some questions.
2. Section A has 18 MCQ’s and 02 Assertion-Reason based questions of 1 mark each.
3. Section B has 5 Very Short Answer (VSA)-type questions of 2 marks each.
4. Section C has 6 Short Answer (SA)-type questions of 3 marks each.
5. Section D has 4 Long Answer (LA)-type questions of 5 marks each.
6. Section E has 3 source based/case based/passage based/integrated units of assessment (4
marks each) with sub parts.

SECTION A (Multiple Choice Questions) Each question carries 1 mark

1. If A ={x∈ C, x2 = 1}, B = {x ∈ C, x4 = 1} then the union of A and B is


a) (-i, i) b) {-1,1} c) {1,-1, i,-i} d) ∅

2. If A is a proper subset B, then (A − B)U(B − A)is equal to


a)(A-B)∩ (B − A) b) A-B c) B-A d) none of these

3. If f: R→ R is defined by f(x)= 3x+|x|, then f(2x) - f(-x) - 6x is equal to


a) f(x) (b) f(2x) c) f(-x) d) –f(x)

4. The solution set of the inequality : 3(2-x)≥ 2(1 − x), if x is an integer


a) (4,∞) b) [4, ∞) c) (−∞, 4] d) (−∞, 4)

5. If ABCD is a cyclic quadrilateral, then the value of cosA+ cosB+ cosC+ cosD.
π
a) 2
b) 0 c)π d) 22π
1+i x
6. If ( ) =1, then
1−i
a) x=2n+1 b) x=4n+1 c) x=2n d) x=4n, n is a natural number.

7. A line passes through the point(2,2) and is perpendicular to the line 3x+y = 3, then the y
intercept of the line is
1 2 4
(a) (b) (c) 1 (d)
3 3 3
8. The equation of the circle concentric with x2+y2-3x+4y-7=0 and passing through
(-1,-2) is
a) x2+y2-3x+4y-1=0 c) x2+y2-3x+4y=0

Page 103 of 119


KVSRO/EKM/XI/MATHS

b) x2+y2+3x-4y-2=0 d) x2+y2+3x-4y=0

9. If a polygon has 44 diagonals, then the number of its sides is


a) 11 b) 7 c) 8 d) none of these

10. The mean deviation of the numbers 3, 4, 5, 6, 7 from the mean is


a) 2.5 b) 5 c) 1.2 d) 0

11. The probability that at least one of the events A and B occurs is 0.6. If A and B occur
simultaneously with probability 0.02, then P ( A ) + P ( B) is
a) 1 b) 0.58 c) 0 d) 0.62

12. Three numbers are chosen from 1 to 20. The probability that they are not consecutive is
187 186 3
a) b) c)190
190 190
d)
1
9
0

13. The mean of 100 observations is 50 and standard deviation is 5. The sum of all squares of
observations is
a) 50,000 b) 250000 c) 252500 d) 255000

x−4
14. If f(x)= 2√x, then f ’(1) is
5 4
a) 4
b) 5
c) 0 d) 4

15. lim(𝑠𝑒𝑐𝑥
𝜋
− 𝑡𝑎𝑛𝑥)is
𝑥→2

a) 2 b) -1 c) 1 d) 0

16. The third term of a G.P is 4. The product of first five terms is
a) 512 b) 1024 c) 40 d) none of these

17. The arithmetic mean of 10 observations is 28. If 3 is added to last number, find the new
arithmetic mean
a) 28.3 b) 31 c) 38 d) 30.3

18.The equation of line passing through (1,1) and parallel to y=3x+4 is


a) y = 3x+2 b) y= 3x+1 c) y= 3x-1 d) y=3x -2

ASSERTION- REASON BASED QUESTIONS


In the following questions, a statement of assertion (A) is followed by a statement of
Reason (R). Choose the correct answer out of the following choices.
(a) Both A and R are true and R is the correct explanation of A.
Page 104 of 119
KVSRO/EKM/XI/MATHS

(b) Both A and R are true but R is not the correct explanation of A.
(c) A is true but R is false.
(d) A is false but R is true.

19. Assertion (A): The expression 6n-5n is always divisible by 25


Reason (R) : The expansion of 6n= 5n+ nCn-1. 5n-1+nCn-2 .5 n-2+ ----- +nC1.5+1

x+2
20. Assertion (A): The limit of f(x) = does not exit at x=2
x2−4
Reason (R) : The function f(x) is not defined at x=2

SECTION B

This section comprises of very short answer type-questions (VSA) of 2 marks each
x
21. Find the domain and range of f(x)=
x+5

OR
sin7x+sin5x
Prove that = −cotx
cos7x−cos5x

22. If A= { 3,5,8}, B={4,6,9,10} and C={3,4,10}, write AX(B-C)

4
23. Find the equation of an ellipse whose vertices are (0,±10) and eccentricity e=
5
24. Find the 12th term of a G.P. whose 8th term is 192 and the common ratio is 2.
OR
Insert 4 geometric means in between 8 and 12.

25. Two students Anil and Ashima appeared in an examination. The probability that Anil will
qualify the examination is 0.07 and that Ashima will qualify the examination is 0.30. The
probability that both will qualify the examination is 0.02. Find the probability that Both
Anil and Ashima will not qualify the examination.

SECTION C
(This section comprises of short answer type questions (SA) of 3 marks each)

26. Find the number of ways in which the word HEXAGON be permuted with vowels together.
OR

Sports team of 7 students is to be constituted, choosing at least 3 from each class XI and
XII. If there are 8 students in each of these classes, in how many ways can the team be
constituted?

Page 105 of 119


KVSRO/EKM/XI/MATHS

27. The longest side of a triangle is twice the shortest side and the third side is 2 cm longer
than the shortest side. If the perimeter of the triangle is more than166cm, then find the
minimum length of the shortest side.
2 2 2 𝑥−𝑦
28. Prove that (cosx-cosy) +(sinx-siny) =4sin ( )
2

29. Find the equation of circle which circumscribes the triangle with vertices (3,4),(3,-6) and
(-1,-2).
OR
Find the coordinates of the foci and the vertices, the eccentricity and the length of the
latus rectum of the hyperbola.
49y2-16x2=784.

30. Find (a+b)4-(a-b)4. Hence evaluate


4 4
(√3 + √2) − (√3 − √2)
OR
𝑥 10
Find the middle term in the expansion of ( + 9𝑦)
3

31. If A= {a,b,c,d,e} , B={a,c,e,g} and C={a,b,g} verify that

A-(B∩ 𝐶) = (𝐴 − 𝐵)𝑈(𝐴 − 𝐶)

SECTION D
(This section comprises of long answer-type questions (LA) of 5 marks
each) ( 4X3=12)

32. The first three numbers in the expansion of (a+b)nare 729, 7290 and 30375. Find the the
values of a, b and n.
OR
If nPr=nPr+1 and nCr = nCr-2, then find the values of n and r.
3
33. Prove that sin20osin40osin60 osin80o=
16
Or
If sinA+sinB=p and cosA+ cos B=q, find the value of sin(A+B) and cos(A+B).
34. Find the derivative of (i) f(x) = xtanxsecx+tanx (3marks)

(ii) f(x)= x2tanx (2 marks)

35. Calculate mean and standard deviation for the following table of age distribution of a
group of people.

Page 106 of 119


KVSRO/EKM/XI/MATHS

Age Number of persons

20-30 3
30-40 51
40-50 122
50-60 141
60-70 130
70-80 51
80-90 2

SECTION E
This section comprises of 3 case-study/passage-based questions of First two case study
questions have three sub-parts (i), (ii), (iii) of marks 1, 1, 2 respectively. The third case
study question has two sub parts (i) and (ii) of 2 marks each.

CASE-BASED/DATA-BASED
36. Read the following passage and answer the questions given below

Conic sections have applications in various fields. When an object is thrown in space,
then the path traced by the object is (called a projectile) a PARABOLA. Another example
can be a parabolic reflectors which are used in Cars, Automobiles ,Solar cookers,
Telescopes and Cables shape like Parabolic arcs used in suspension bridge. The Planets in
a solar system moves in an ELLIPTICAl path with the Sun at one of the foci, also Artificial
satellites are made to move in an ELLIPTICAL path around Earth. HYPERBOLA have their
applications in the field of Ballistics, the shape of EIFFEL Tower is also Hyperbolic .

i) What is the length of latus rectum of y2=-3x (1 mark)


ii) Write the equation of an equilateral hyperbola with length of transverse axis is
12cm . (1 mark)
iii) A man is running a race course in such a way that that the sum of his distances
from two flag posts is always a constant and it is 10m. if the distance between two
flag posts is 8m, what is the locus of path traced by him? Write the equation.
(2 marks)

Page 107 of 119


KVSRO/EKM/XI/MATHS

37. The importance of games and sports in a student’s life is to a great extent. It has proved to
be very therapeutic in nature. Sports in a student’s life helps in improving and building
skills, and also aid in other concepts such as dispute management and sport-based
interaction. Sports and games not only improve physical and mental health they also
inculcate discipline in students. They learn to work in teams and their self-confidence
gets a boost.

A Physical Education teacher conducted a survey of 500 students to study about


their interest in various sports and games. It was found that 285 are interested in
playing football, 195 are interested in playing basket ball, 115 are interested in playing
hockey. 45 are interested in playing football and hockey, 70 are interested in playing
football and basket ball and 50 are interested in playing hockey and basket ball. 50
students are not interested any of these three games.
i.How many students are interested in at least one of the three games?
(1 mark)
ii. How many are interested in all three games? (1 mark)
iii. How many are interested exactly in one game? (2 marks)

38. The number lock has three/four wheels each labeled with 10 digits 0 to 9.The lock can be
opened with the digits are set in a particular specific order. In mathematics, a
permutation of a set is an arrangement of its members into a sequence or linear order, or
if the set is already ordered, a rearrangement of its elements. The word "Permutation"
also refers to the act or process of changing the linear order of an ordered set.

Page 108 of 119


KVSRO/EKM/XI/MATHS

Now answer the questions given below

(i) A number lock in a suitcase has three wheels each labeled with ten digits 0 to 9
and the number lock is a three digit even number, then find the number of all possible
attempts if repetition of numbers is not allowed.
(2 marks)
(ii) Find the number of unsuccessful attempts if repetition of digits is allowed.
(2 marks)

MARKING SCHEME
SAMPLE PAPER 1
Qn. No Answer mark

SECTION A

1 Option c: {1,-1,I,-1} 1mark

2 Option c: B-A 1mark

3 Option a: f(x) 1mark

4 1mark
Option b: [4, ∞)

5 Option b: 0 1mark

6 Option d: 4n+1 1mark

7 Option d: 4 1mark
3
8 Option c: x2+y2-3x+4y=0 1mark

9 Option a: 11 1mark

10 Option c: 1.2 1mark

11 Option d: 0.62 1mark

187
12 Option a: 1mark
190
13 Option a: 50000 1mark

14 Option a: 5 1mark
4

Page 109 of 119


KVSRO/EKM/XI/MATHS

15 Option d: 0 1mark

16 Option b: 1024 1mark

17 Option a: 28.3 1mark

18 Option d: 3x-2 1mark

19 Option d: A is false but R is true 1mark

20 Option b: A is true but R is not the correct explanation


of A. 1mark

SECTION B

21 Domain R-{-5} Range R-{1} 1mark each

OR
sin7x+sin5x 2sin6x cosx
= 1
cos7x−cos5x −2sin6xsinx 1 mark
2
= cotx
1 mark

B – C= {6, 9} 1mark
22 Ax(B-C)= {(3,6), (3,9),{5,6), (5,9),(8,6),(8,9)} 1 mark

a=10 , c=ae=8, b=√𝑎2 − 𝑐2= 6 1 mark


y2 2
23
Equation of ellipse : +x =1
100 36 1 mark
3 1 mark
A.27=192. A=
24 2
3
T12 = x2 =3072
11 1mark
2
OR
5 3
24 8xr5=12,, r= √ 1 mark
2
5 3 5 9 5 27 5 81
4 G Ms are 8 √ , 8 √ , 8 √ , 8 √ 1 mark
2 4 8 16
25 P(atleast one of them pass the examination) 1 mark
=0.07+0.3-0.02=0.35
P(both of them will not qualify the exam)=1-0.35=0.65
1 mark

SECTION C

Page 110 of 119


KVSRO/EKM/XI/MATHS

The number of ways vowels can be permuted=3!=6 1mark


26 The number of ways remaining letters and 3 vowels together=5!x
3!=120x6=720
OR 2 mark
The number of ways of section of 3 students from class XI and 4
students from class XII = 8C3x8C4 = 3920 1 mark
The number of ways of section of 4 students from class XI and 3
students from class XII = 8C4x8C3 = 3920
Total number of ways=2x3920=7840 1 mark
1 mark

Let the shortest side be xcm 1


mark
27 2
Then longest side is 2x cm 1
mark
2
Third side is x+2 cm 1
mark
4x+2≥ 166, 𝑥 ≥ 41𝑐𝑚
2
1 mark
Minimum length of shortest distane is 41 cm.
1
mark
2
𝑥+𝑦 𝑥−𝑦
28 Cosx-cosy =-2sin )sin ) 1 mark
( 2
( 2
𝑥+𝑦 𝑥−𝑦
Sinx-siny = 2 cos( 2 ) 𝑠𝑖𝑛 ( 2 ) 1 mark
𝑥−𝑦
(𝑐𝑜𝑠𝑥 − 𝑐𝑜𝑠𝑦)2 + (𝑠𝑖𝑛𝑥 − 𝑠𝑖𝑛𝑦)2 = 4𝑠𝑖𝑛2 )
(
2 1 mark

Let the equation of circle be x2+y2-2hx-2ky+c=0 1


mark
29 2
Substitute the points to get equations : 25-6h-8k+c=0
45-6h+12k+c=0
1 mark
5+2h+4k+c=0
Solve the equations to get h=4,, k=-1 and c=-9
Equation of circle is x2+y2-8x+2y-9=0 1 mark
OR 1
mark
𝑦2 𝑥2 2
Equation of hyperbola : − =1
16 49
√65
Vertices(0,±4), foci(0,±√65), e= 1
4
, mark
49 2
Length of Latus Rectum= =24.5
2 2 mark
1
mark
2

(a+b)4-(a-b)4=2(4a3b+4ab3) 2 mark
4 4
(√3 + √2) − (√3 − √2) = 24√6 +16√6=40√6 1 mark

Page 111 of 119


KVSRO/EKM/XI/MATHS

30 OR
x 5
Middle term is T = 10C ( ) (9𝑦)5=18x14x243(xy)5=61,,236(xy)5
6 5
3
3 marks

31 B∩ 𝐶 = {𝑎, , 𝑔} 1
mark
2
A-(B∩ 𝐶) = {𝑏, 𝑐, 𝑑, 𝑒} 1
mark
2
A-B={b,d} and A-C ={c,d,,e}
1 mark
1
(A-B)U(A-C)= {𝑏, 𝑐, 𝑑, 𝑒} mark
2
A-(B∩ 𝐶)=(A-B)U(A-C)
1
mark
2

SECTION D

𝑛(𝑛−1)𝑎 𝑛−2 𝑏2 1
32 The expansion of (a+b)n=an+nan-1b+ +…. mark
2
2
𝑛(𝑛−1)𝑎 𝑛−2𝑏2
an = 729, nan-1b=7290 , = 30375
2 1
After simplification of these equations 1 mark2
a n
=
b 10
n 7290x7290 6
= =
n−1 2x30375x729 5 2 marks
N=6,, b=5 and a=3
OR 1 mark
n! n!
=
(n − r)! (n − r − 1)!
n! n!
= 1 mark
r! (n − r)! (n − r + 2)! (r − 2)!

After simplification, n-r =1 and r(r-1)=6. r=3, n=4 1 mark

3 mark

3
33 LHS =√ sin80sin20sin40 1 mark
2
−√3 (cos100 − cos60)sin40
= 1mark
4
−√3 (sin140 − sin60 − sin40)
=
8
−√ 3 √3 3 1 mark
= (2cos90sin50 − )= 2mark
8 2 16

Page 112 of 119


KVSRO/EKM/XI/MATHS

OR

A+B A−B
2sin( ) cos ( )=p
2 2
A+B A−B 1 mark
2cos( ) cos ( )=q
2 2

A+B p A+B p A+B q


tan( )= , sin( )= , cos( )=
2 q 2 √p2+q2 2 √p2+q2
2 mark
2pq p2−q2
sin(A+B)= , cos(A+B)= 2 mark
p2+q2 p2+q2

34 i) f (x) = tanx(xsecx+1)
f ‘ (x)=sec2x(xsecx+1)+ tanx(secx+xsecxtanx) 3 mark
Ii) f(x)= x2tanx
f ‘ (x) =x2sec2x+2xtanx 2 mark

1
35 A=55, h=10, ∑ 𝑓ⅈ 𝑢ⅈ = 5, ∑ 𝑢ⅈ 2𝑓ⅈ =705 2 mark
2

A. M=55.1 1 mark
1
1 mark
2
Variance= 140.99, Standard deviation=11.87
SECTION E
36 i) Length of latus rectum= 3 1 mark
ii) x2-y2=36 or y2-x2= 36 1 mark
X2 Y2
Locus is an ellipse + =1
25 9 2 mark

37 I) 450 students like atleast one game 1 mark


ii) 20 students like all three games. 1 mark
iii) 325 like exactly one game 2 mark

38 i) 8x8x5=320 2mark
ii) 9x10x5-1=449 2mark

Page 113 of 119


KVSRO/EKM/XI/MATHS

Sample Question Paper 2


CLASS-XI 22- 23
MATHS BLUE PRINT

MCQ A&R 2 3 5 C.S


SI CHAPTER WEITAGE
(1) (1) MARK MARK MARK 4
1 SETS 3 1
2 RELATIONS AND FUNCTIONS 2 1 23
3 TRIGNOMETRIC FUNCTIONS 2 1 1 1
COMPLEX NUMBERS AND
4 2
QUADRATIC EQUATIONS
5 LINEAR INEQUALITIES 1 1
PERMUTATIONS AND 25
6 2 1
COMBINATIONS
7 BINOMIAL THEOREM 1 1
8 SEQUENCES AND SERIES 1 1 1
9 STRAIGHT LINES 1
10 CONIC SECTIONS 1
12
INTRODUCTION TO THREE
11 1 1
DIMENSIONAL GEOMETRY
12 LIMITS AND DERIVATIVES 3 1 1 8
13 STATISTICS 1 1
12
14 PROBABILITY 1 1 1
NUMBER OF QUESTIONS 18 2 5 6 4 3
TOTAL MARKS 18 2 10 18 20 12 80

Sample Question Paper 2


Class XI
Session 2022-23
Mathematics

Time Allowed: 3 Hrs Maximum Marks: 80

General Instructions :

1. This Question paper contains - five sections A, B, C, D and E. Each section is


compulsory. However, there are internal choices in some questions.
2. Section A has 18MCQ’s and 02 Assertion-Reason based questions of 1mark each.
3. Section B has 5 Very Short Answer (VSA)-type questions of 2 marks each.
4. Section C has 6 Short Answer (SA)-type questions of 3 marks each.
5. Section D has 4 Long Answer (LA)-type questions of 5 marks each.
6. Section E has 3 source based/case based/passage based/integrated units of
assessment (4 marks each) with sub parts.

Page 114 of 119


KVSRO/EKM/XI/MATHS

SECTION A
(Multiple Choice Questions)
Each question carries 1 mark

1. Write the solution in roster form A= {x: x∈ R, 𝑥2-3x- 40=0}

(a) {8,-5} (b) {-8,5} (c) {10,-4} (d) {-10,4}

2. In a school there are 20 teachers who teach mathematics or physics. Of these,12 teach mathematics
and 4 teach both physics and mathematics. How many teach physics ?

(a) 4 (b) 8 (c) 12 (d) 16

3. If(x/3 ,y-1) =(3,7) find x,y

(a) 9,8 (b) 32,4,43 (c) 16,45 (d) 64,54

4. Range of signum junction is

(a) [0,∞) (b) (-∞, ∞) (c) {-1,0,1} (d) (-∞, 0]

5. If A= {a, b, c, ∅} ,then numbers of subsets A has

(a) 4 (b) 16 (c) 32 (d) 8

6. cosec (– 1410°)

(a) -1 (b) 0 (c) 1 (d) 2


𝜋 𝜋
7. 𝑐𝑜𝑡2 + co sec 5𝜋 + 3𝑡𝑎𝑛2 = ?
6 6 6

(a) 6 (b) 10 (c) 3/2 (d) -1/2

8. Solve : 3x – 7 > 5x – 1

(a) [2,∞) (b) (−∞ ,4] (c) (−∞, 4) (d) [4,∞)

9. 3(7 + i7) + i (7 + i7)

(a) 14+28 i (b) 28+14 i (c) 14-28 I (d) -14+28 i

10. How many 3-digit even numbers can be formed from the digits 1, 2, 3, 4, 5, 6
if the digits can be repeated?

(a) 216 (b) 120 (c) 108 (d) 72

11. Find the multiplicative inverse of4 – 3i

(a) 4+3 i (b) 4 +i 3 (c) 4 - 3 i (d) 4 -i 3


25 25 25 25

12. How many chords can be drawn through 21 points on a circle?

Page 115 of 119


KVSRO/EKM/XI/MATHS

(a) 200 (b) 180 (c) 420 (d) 210

13. Find the 4th term in the expansion of(𝑥 − 2𝑦)12

(a) -1760 𝑥9𝑦3 (b) 1760 𝑥3𝑦9 (c) .760 𝑥9𝑦3 (d) -760 𝑥3𝑦9

14. If the sum of a certain number of terms of the A.P. 25, 22, 19, … is 116. Find
the last term

(a) 3 (b) 4 (c) 2 (d) 6

15. Find the distance between the points P(1, –3, 4) and Q (– 4, 1, 2)

(a) 3 √5 (b) 2 √5 (c) 45 (d) 35


𝑥15−1
16. Evaluate : lim -
𝑥→1 𝑥10 −1

(a) 2 (b) 2/3 (c) 3 (d) 3/2

17. Find the limits lim 𝑥3 −2𝑥2


𝑥→2 𝑥2 −5𝑥+6

(a) -4 (b) 4 (c) 3 (d) -2


𝑠ⅈ𝑛 4𝑥
18. Evaluate lim
𝑥→0 𝑠ⅈ𝑛2𝑥

(a) 3 (b) 1 (c)2 (d) 4

Assertion Reason Based Questions

In the following questions , a statement of assertion (A) is followed by a statement of Reason (R) .
Choose the correct answer out of the following choices.

(a) Both A and Rare true and R is the correct explanation of A.


(b) Both A and R are true but R is not the correct explanation of A.
(c) A is true but R is false.
(d) A is false but R is true.

19. Assertion (A): Range of relation R={(x,y) : x,y∈ N, y = x2+3, 0<x<5} is {4, 7, 12, 19}
Reason (R) : The set which contains all the second elements, on the other hand, is known as the range
of the relation

20. Assertion (A): If A is the set of even natural numbers less than 8 and B is the set of prime
numbers less than 7, then the number of relations from A to B is 29
Reason (R) : If set A has P elements and set B has q elements then the number of relations from A to B
is 2𝑝𝑞

Page 116 of 119


KVSRO/EKM/XI/MATHS

Section B
(This section comprises of very short answer type questions (VSA) of 2 marks each)

21. Let A = {1, 2}, B = {1, 2, 3, 4}, C = {5, 6} and D = {5, 6, 7, 8}. Verify that
(i) A × (B ∩ C) = (A × B) ∩ (A × C).
OR
Let A = {9,10,11,12,13} and let f : A→N be defined by f (n) = the highest prime
factor of n. Find the range of f.

22. If in two circles, arcs of the same length subtend angles 60° and 75° at the
centre, find the ratio of their radii

23. How many terms of the A.P. – 6, -11/2, – 5, … are needed to give the sum –25?
OR
Insert five numbers between 8 and 26 such that the resulting sequence is an A.P
𝑐𝑜𝑠 2𝑥 −1
24. Evaluate lim
𝑛→0 𝑐𝑜𝑠 𝑥 −1

25. In an entrance test that is graded on the basis of two examinations, the probability of a randomly
chosen student passing the first examination is 0.8 and the probability of passing the second
examination is 0.7. The probability of passing at least one of them is 0.95. What is the probability of
passing both?

Section C
(This section comprises of short answer type questions (SA) of 3 marks each)
𝑐𝑜𝑠4𝑋+𝑐𝑜𝑠3𝑋+𝑐𝑜𝑠2𝑋
26. = cot 3x
𝑠ⅈ𝑛4𝑋+𝑠ⅈ𝑛3𝑥+𝑠ⅈ𝑛2𝑥

27. In how many ways can one select a cricket team of eleven from 17 players in which only 5 players
can bowl if each cricket team of 11 must include exactly 4 bowlers?
OR
Determine the number of 5-card combinations out of a deck of 52 cards if each selection of 5 cards
has exactly one king

28. 1 × 2 + 2 × 3 + 3 × 4 + 4 × 5 +...
OR
How many terms of G.P. 3, 32 , 33 , … are needed to give the sum 120?

29. Find the equation of a line that cuts off equal intercepts on the coordinate axes and passes through
the point (2, 3).
OR
Find equation of the line parallel to the line 3 x-4y+2=0 and passing through the point (–2, 3).

30. Find the ratio in which the YZ-plane divides the line segment formed by joining the points (–2, 4, 7)
and (3, –5, 8).

31. Find the derivative of cos x from first principle.

Page 117 of 119


KVSRO/EKM/XI/MATHS

Section D
(This section comprises of long answer type questions (LA) of 5 marks each)

Find the general solution cos 3x + cos x – cos 2x = 0


OR
Prove that 𝑐𝑜𝑠2 x + 𝑐𝑜𝑠2(x+𝜋)+𝑐𝑜𝑠2 (x- 𝜋) = 3
3 3 2

33. A manufacturer has 600 litres of a 12% solution of acid. How many litres of a 30% acid solution
must be added to it so that acid content in the resulting mixture will be more than 15% but less than
18%?

34. The coefficients of three consecutive terms in the expansion of (1 + a) n are in the ratio1: 7 : 42.
Find n.
OR
Prove that the coefficient of 𝑥 in the expansion of(1 + 𝑥)2𝑛 is twice the coefficient of 𝑥𝑛 in the
𝑛

expansion of (1 + 𝑥)2𝑛−1 .
2 2
35. Find the foci, vertices , eccentricity and length of latus rectum of the ellipse 𝑥 +𝑥 =1
36 16

Section – E
(This section comprises of 3 case study / passage - based questions of 4 marks each with two
sub-parts.First two case study questions have 3 sub parts (i),(ii),(iii) of marks 1,1,2
respectively.The third case study question has two sub parts of 2 marks each.)

36. In a survey of 50 persons of an apartment, it was found that 15 persons read magazine A, 16
persons read magazine B, 16 persons read magazine C, 8 read both A and B, 10 read both B and C, 7
read both C and A, 5 read all the three magazines.
(a) How many persons only read magazine A.
(b) How many persons read only magazine C.
(c) How many read at least one of the magazines.
37. Mohan is doing one of his project. For this he asked shoe size of 10 of his class mates which are as
follows-
6,5,5,6,8,6,7,7,8,8

(a) What would be the mean shoe size for the data?
(b) What would be the median for the data?

(c) What would be the mean deviation about mean for the data?

38. A bag contains 20 white slips, 10 red slips, 40 yellow slips and 10 blue slips. If these 80 slips are
thoroughly shuffled so that each slip has the probability of drawing a slip of paper that-
(a) White
(b) Blue or White

Page 118 of 119


KVSRO/EKM/XI/MATHS

Scoring Key

1) (a) {8,-5} 20) (a)


2) (c) 12 21) - or {3,5,11,13}
3) (a) 9,8 22) 5:4
4) (c) {-1,0,1} 23) 5 or 20
5) (b) 16 24) 4
6) (d) 2 25) 0.55
7) (a) 6 26)
8) (b) (- ∞,4] 27) 3960 or 4c1 X 48c4
9) (a) 14+28c° 28) n/3(n+1)(n+2)
10) (c) 108 29) 4
11) (b) 4/25+i3/25 29)) x+y=5 or 3x-4y+18=0
12) (d) 210 30) 2:3
13) (a) -1760x^9y^3 31) –sinX
14) (b) 14 32) x= (2n+1)π/4 or
2nπ+π/3 nE Z
33)120<x<300
15) (a) 3√5 34) n=55 or
16) (d) 3/2 35) F=(+- √20,0)
V=(+-6,0)
E=√20/6 L=16/3
17) (a) -4 36) a) 5
b) 4
c) 27
18) (c) 2 37) a)6.6 b) 6.5 c)1
19) (a) 38) (a)¼ (b)3/8

Page 119 of 119

You might also like